SlideShare a Scribd company logo
1 of 31
Download to read offline
1
Ministry of health of Ukraine
Vinnytsya national Pirogov memorial medical university
APPROVED
on the methodical conference of department
of endoscopic and cardiovascular surgery
of medical faculty №1
Head of Department
Professor ___________ V.V.Petrushenko.
«______» ___________________20___
METHODICAL RECOMMENDATIONS
for students
Educational discipline Surgery
Module № 2
Informative module 3
Theme of classes Arterial thrombosis and embolism.
Classification of acute limb ischemia.
Clinical stages of motion. Diagnostic.
Differential diagnosis Methods of
surgical treatment.
Course V
Department Medical №1
Vinnytsya 2016
2
Actuality
Vascular diseases of extremities are widespread enough pathology with that
in the first turn, surgeons, domestic and district doctors, and also the doctors of
urgent first-aid have business mostly. Arterial thrombosis and embolisms of main
vessels are a division of urgent surgery of vessels, which until now is to perfection
an unsolved problem. It and large death rate (from 20 to 35%), large frequency of
amputations of extremities on soil of gangrene, loss of capacity, a considerable exit
on disability of patient, that worsens quality of life. The timely diagnostician of
sharp arterial ischemia, reduction of time for diagnostics and grant of skilled
medical care is stipulated by the economies of extremities and considerably
improve the social problems of patient.
Educational objectives:
- to teach students to recognize the symptoms of acute arterial ischemia;
- to able to differentiate embolism and thrombosis of blood vessels;
- to be able to determine the location of vascular lesions;
- to teach students to choose their own method of operative surgery;
- to know the methods of surgical treatment of thrombosis, embolism;
- to conduct necessary conservative therapy (thrombolytics, anticoagulants,
antiplatelet agents, antispasmodics, etc.).
The student should to know:
- arterial and venous circulatory system;
- to determination of acute arterial ischemia;
- causes thrombosis;
- morphological tissue changes in acute arterial obstruction;
- reasons for the development of acute arterial ischemia;
- to definition and causes of arterial embolism;
- classification of acute arterial ischemia (V.S.Savelyev, 1978);
- symptoms and clinical course of acute limb ischemia;
- variants of clinical course and complications;
- diagnostic algorithm examination of the patient;
- differential diagnosis of thrombosis, embolism, spasm of the arteries of the
extremities;
- tactics and choice of treatment.
The student should to be able to:
- to identify pulsation in the extremities;
- to identify violations of blood circulation;
3
- to diagnose the causes of artisanal embolism and thrombosis;
- to identify the clinical signs of acute arterial ischemia;
- to interpretation pursue complementary examination methods (coagulation,
vascular ultrasound, angiography of blood vessels);
- to able to determine the location of thrombus and embolism;
- to be able to formulate a diagnosis violation artisanal circulation;
- to conduct a differential diagnosis of thrombosis, embolism, spasm of the
arteries of the extremities;
- to be able to work out the tactics of choice of surgical treatment;
- to hold anticoagulation or thrombolysis.
Table of contents of theme
Sharp arterial impassability
Sharp arterial impassability believe the sudden cessation of blood circulation in
the main artery embolism as a result of thrombosis or against other diseases.
Etiology and pathogenesis
In 80-93% of cases the cause arterial embolism have heart disease, including
atherosclerotic infarction, acute myocardial infarction, rheumatic mitral defects
like.
When arterial embolism of the extremities due to lack of time the possibility of
development of collateral circulation is always limited.
Acute thrombosis prerequisites for enhancing collateral circulation in most
cases formed during the progression of the underlying disease (atherosclerosis,
arteritis, etc.). It also helps to increase the flow of arterial occlusion proximal level.
Causes intravascular thrombus R. Virhov (1856), which united them-known
triad: damage to the vascular wall, changes in the blood and circulatory disorders.
In 90% of cases the cause of arterial thrombosis is "arteriosclerosis".
Pathomorphism
In patients with acute arterial obstruction in the first minute of watching
histochemical and ultrastructural changes in tissue that indicate their ischemia: no
cytochrome, succinate dehydrogenase, glycogen and degradation of existing
mitochondria. In the future, there are degenerative, necrobiotic, necrotic processes,
and infarcts, hemorrhage and gangrene.
Classification of acute ischemic limb
The classification is based by V.S.Savelyevym (1973) laid the clinical course
of the disease (table 1).
4
Table 1
The degree of limb ischemia The main feature of
IS (ischemic stress) No symptoms of ischemia at rest (their
appearance only at load)
I A Numbness, cold, paresthesia
I B Pain
II A Paresis
II B Plehiya
III A Subfastsialnyy swelling muscles
III B Partial contracture
III C Total contraction
Symptomatology and clinical motion
Subjective symptoms of acute ischemic limbs. The pain in the affected limb in
most cases is not the first sign of acute arterial obstruction. Especially when he is
pronounced embolism. The beginning of the disease in these cases is always
sudden, although the power of pain can be different. However, acute thrombosis of
major arteries syndrome pain in most cases develops gradually. In the initial stages
of its intensity expressed slightly, then it increases, and thereafter it may no
different from similar embolism. The nature of this syndrome is primarily caused
by low arterial spasm and depends on the extent of ischemia tissue.
Feeling of numbness, drop in temperature, paresthesia appear for patients both
at embolism and at a sharp thrombosis. They usually grumble about feeling of
"crawl of ants", passing of electric current" in extremity, "pricking needles". The
degree of expression of the indicated complaints often depends on the
psychological equilibrium of patients.
Evidence of acute ischemic limbs. Change the color of the skin. At the onset of
the disease observed moderate pale skin. Then it joins cyanosis, which is at a much
severe ischemia causes the "marbled pattern" skin.
Reducing the skin temperature celebrate almost 100% of cases. Its fluctuations
acquire a wide range, so always have to measure the temperature in the symmetric
parts of sick and healthy limb.
Sensitivity disorders also constantly accompany acute ischemia of the limbs. In
the initial stages of arterial occlusion reduced the surface, and then, with an
increase ischemia, and deep sensitivity. During prolonged ischemia and significant
comes complete anesthesia.
Violations of active movements in joints limbs are characteristic of ischemia.
They can occur either as constraints (paresis), or absence (plehiya) active
movements or first in the distal and later in the proximal joints.
Tenderness ischemic muscle indicates unfavorable course of acute ischemic
5
limbs and usually occurs in severe forms.
Subfastsialnyy swelling of the muscles occurs when long-standing, long, very
severe ischemia. It mainly "tight" and does not extend above the knee joint and the
feet above the elbow to the hand.
Coronary heart muscle contracture usually indicates the beginning necrobiotic
phenomena in limbs. There are partial (partial) individual contracture of joints in
which passive movements are not possible, and try to active movements painful,
and total (complete) contracture, in which the movement is not possible in all
joints.
Clinical manifestations of ischemia are most pronounced in the distal limb.
Their proximal limits are always below the occlusion, but define it in many cases
impossible.
Variants of clinical motion and complication
Acute occlusion of the aortic bifurcation is characterized by clinical signs of
ischemia both extremities, anterior abdominal wall, pelvic and distal spinal cord.
Acute occlusion of the femoral artery, usually develops in the origin of the deep
artery of the thigh. The upper limit is poor circulation between the upper and
middle thirds of the thigh. In all cases, time has not treated the disease movements
in the ankle joints of fingers and feet are missing, and only 28-30% of patients with
this symptom can be observed in the knee joint.
The sharp occlusion of popliteal artery mostly arises up at the level of
trifurcation, that is on a 6-7 cm below poples. The top limit of violation of
sensitiveness is no communicative between overhead and lower one third of shin.
The changes of agile functions, as a rule, will keep indoors the joint of tibial foot
for an area.
For the occlusion of arteries of overhead extremities characteristic is that a
sharp ischemia arises up in young age (35-40 years) and in default of any diseases
of heart. In anamnesis on occasion it is possible to reduce the phenomenon of
plexites or neck-pectoral radiculitis, quite often at the inspection of supraclavicular
area find out smoothing or bulgingof supraclavicular fossula at palpation of that
determine dense education with a pulsation.
Classification of variants of motion of sharp ischemia of extremity
Table 2
A sharp increasing
ischemia of extremity
A sharp ischemia of
extremity is with the
moderate stabilizing
A sharp ischemia of
extremity is with progress
Ischemia of tension Ischemia of calmness
With
slow
With rapid
growth
Quick as
lightning
With slow progress With rapid
progress
6
growth
Gangrene Chronic arterial insufficiency
Semiotics angiographic embolism, breakage of contrast in the area of
bifurcation of the arteries, "poor" net of collateral (Pict. 1).
Angiographic semiotics of acute thrombosis, placed crooked line breakage
contrast, uneven, jagged contours on fonf artery atherosclerotic vascular lesions
and well-developed net of collateral (Pict. 2).
Pict. 1. Embolism of femoral Pict. 2. Thrombosis of right
artery. Arteriogram. A pointer general iliac artery. Arteriogram.
is indicate localization of A pointer is indicate
embolus. localization of blood clot.
7
Diagnostic program
1. Complaints,anamnesis.
2. Review of extremity, estimation of viability of ischemic tissues.
3. Palpation, auscultation of arteries.
4. Dopplerographic of inspection.
5. Aortoarteriohrafiya.
6. Coagulogram.
The gangrene of extremity, that is necessary to be considered the most
dangerous complication with a poor prognosis, develops in case of the unliquidated
sharp arterial impassability.
Differential diagnosis
Table 3
Diagnostic sign Embolism Sharp thrombosis Spasm
Basic disease Rheumatic defeats
of heart, blinking
arrhythmia.
Atherosclerosis,
trauma, malignant
tumors.
Heart disease,
trauma, acute
thrombophlebitis.
Beginning Acute Subacute Acute
Pain Very intense Mild Intense
Ischemia Sharply expressed Moderate Moderate
Skin sensitivity Anesthesia Reduced Reduced
Motor function Ischemic paralysis Reduced muscle
strength
Reduced muscle
strength
Skin temperature Sharply reduced Moderately
reduced
Moderately
reduced
The color of the
skin
Pale, almost white
or marble
Pale Pale purple tint
Swelling of limbs Sometimes, after
first aid and only
shin
Do not have There is early in
acute venous
thrombosis
8
Angiography A clear contrast
level cliff
The contours of
the vessel unequal,
line break
Vessels with
smooth
disappearance of
contrast
Collaterals Missing A large number of Collaterals of
narrowed diameter
Tactics and choice of treatment
In acute arterial obstruction indications for conservative treatment there can
only be a contraindication to surgery.
Absolute contraindications are: agonal condition of the patient, total limb
ischemic contracture (acute ischemia degree IPR), a very serious condition patient
with mild ischemia (acute ischemia IA - IB degrees).
By relative contraindications should include acute myocardial miokardachy
stroke, tumors that are not subject to the operation, and easy ischemia without
signs of progression.
In all other cases of acute arterial obstruction appropriate emergency surgery. The
optimal time for surgery for embolism is the first 6 hours of onset and acute
thrombosis - 12-24 hours.
The essence of surgery (Pict. 3) embolism is emboli generally remove the
catheter, vascular rings, vacuum suction, by retrograde flushing the arteries
Pict. 4), or spurly balloon probe. The most famous among them, recently received
a special method embolectomy balloon catheter by Fogarty. Its use has made low-
impact operation, simple and much more effective.
Surgeries acute thrombosis fundamentally different from dealing with
embolism. Features of surgical intervention in patients with acute thrombosis is
that while thrombectomy should also eliminate the cause of the disease, that
performs particular reconstruction arteries.
In uncomplicated postoperative patients can prescribe outpatient treatment for
12-15 days. In the future, they should be at the dispensary under the control of the
surgeon, and if necessary - and cardiologist and rheumatologist. After undergoing
reconstructive surgery on main arteries they must also conduct periodic state
determination coagulation of the blood system. Such patients should receive
aspirin regularly and trental. If necessary, we must purposefully use indirect
anticoagulants with obligatory once in six months, a full examination of the patient
in the clinic or hospital conditions.
9
A B
C D
Pict. 3. Removal of emboli (scheme):
A - stream of blood; B - tweezers; C - Fogarty catheter; D - vacuum suction.
Pict.4. Scheme of operation of the retrograde washing of artery force-feeds.
Tests and tasks are added.
10
Theme: The arterial thrombosis and embolisms
Tests (Step-2)
Variant 1
1. The patient, 35 years old, from rheumatism in the active phase has mitral
valve stenosis fourth stage, atrial fibrillation. Six hours ago the expressed
pains appeared at a calmness in a right lower limb, at an objective review the
arterial pulsation of femoral artery is absent on the right, the temperature of
skin of foot is mionectic, bluish color, motion limits in a foot, a sensitiveness
is stored. Define illness.
A. Sharp violation of cerebral circulation of blood
B. Sharp right femoral artery thromboembolism
C. Sharp thrombophlebitis of deep veins of right shin
D. Erysipelas right tibia
E. Lymphangoitis acute right lower limb
2. A patient with the sharp ischemia of both lower limbs, that arose up
simultaneously, absent pulsation on right and left femoral, popliteal arteries.
Great pain appeared in a stomach, vomit, collapse, liquid chairs, positive
symptoms of irritation of peritoneum. What is the condition of the patient?
A. Occlusion of right iliac artery
B. Thrombosis and embolism aortic bifurcation inferior mesenteric artery
C. Thrombosis of both femoral arteries
D. Layering aortic aneurysm
E. Total thrombosis of bifurcation of aorta
3. The man complained of pain in the left arm, pain at bending in elbow and
radial-wrist joints. Pains appeared 12 hours ago after heavy physical activity.
Objectively: the edema of the left brush, forearm, shoulder is expressed.
Skin on the left limb cyanotic, reduced pain and tactile sensitivity. Pulse on
the hand, forearm and shoulder is not defined.
Formulate a preliminary diagnosis:
A. Sharp thrombosis of the subclavian vein
B. Sharp thrombosis of humeral artery
C. Syndrome by Reyno
D. Illness by Reyno
E. Break of armpit artery
11
4. The patient, 56 years old, complaining of intense pain in the left leg, there
were three hours ago suddenly, numbness and cold extremities. During the
year, noted the pain in the leg while walking, increased sensitivity to
cooling. The left foot and lower leg with marble pattern, marked "a
symptom of the groove." At the touch of foot cold reduced active movement
of the foot, toes saved. Pulse detectable only in the femoral artery. Above
her systolic murmur is heard. Formulate a preliminary diagnosis:
A. Endarteritis obliterans
B. Syndrome by Reyno
C. Sharp thrombophlebitis
D. Atherosclerosis of vessels of lower limbs, sharp thrombosis
E. Illness by Buerger
5. The patient, 62 years old, complaining of intense pain in the left leg, there
were three hours ago suddenly, numbness and cold extremities. During the
year, noted the pain in the leg while walking, increased sensitivity to
cooling. The left foot and lower leg with marble pattern, saphenas slept. At
the touch of foot cold reduced active movement of the foot, toes saved.
Pulse detectable only in the femoral artery. Above it rough systolic murmur
is heard. Formulate a preliminary diagnosis:
A. Endarteritis obliterans
B. Stenosis of the left popliteal artery
C. Sharp occlusion of the left femoral artery
D. Sharp thrombophlebitis
E. Sharp ileofemoralnyy arterial thrombosis
12
Theme: The arterial thrombosis and embolisms
Tests (Step-2)
Variant 2
1. In patients with acute ischemia of the extremities of both encountered while,
but has more significant manifestations of matter, there is no ripple in the
right femoral, popliteal arteries and the arteries of the foot. Determined
relaxation ripple left thigh arteries and other arteries of the left lower limb.
Determine the level of occlusion:
A. Occlusion of the right iliac artery
B. Tromb - "rider" aortic bifurcation
C. Occlusion of both femoral arteries
D. Forensic thrombosis aortic bifurcation
E. Occlusion of the right external iliac and femoral arteries of the left
internal
2. The patient, 45 years old, is on treatment for rheumatism, active phase,
combined mitral valve defect. During the morning the toilet suddenly felt
pain in his left hand, followed her numbness. Pain and numbness are grew.
Objective: pale skin of his left hand, relatively cold. Pulsation of the arteries
throughout the hand is missing.
What is the most appropriate therapeutic approach?
A. Appointment fibrinolysis, anticoagulants
B. Purpose of antibiotics and anti-inflammatory drugs
C. Emergency embolectomy
D. Probing heart
E. Emergency trombintymektomiya
3. In the district hospital which is located 30 km from the central district
hospital and 180 km from the regional hospital, hospitalized patient is 62
years of intramural myocardial infarction. After 25 days of onset of night he
suddenly appeared sharp pain in the left lower limb. Ending immediately
became pale. Gone active movement of toes and ankle-stage joint, decreased
skin sensitivity to palpation of the calf muscle moderately painful. Ripple in
the hip during inguinal artery connection, left - enhanced, compared with
femoral artery pulsation right. What is the most likely diagnosis?
13
A. Thrombosis of the femoral artery
B. Atherosclerosis of the arteries of the lower extremities
C. Embolism femoral artery
D. Illness by Reyno
E. Endarteritis obliterans of the lower extremities
4. The patient, 60 years old, there was a sharp pain in the left lower limb,
which appeared within hours and growing. Vascular Angiography revealed
that the contours of the popliteal artery uneven, irregular line break, plenty
of collaterals. What is the status of the patient?
A. Popliteal artery embolism
B. Trumbo popliteal artery
C. Spasm of the popliteal artery
D. Illness by Reyno
E. Tthrombophlebitis
5. The patient came to the clinic 3 hours ago with the onset of pain in his right
leg. Sick 10 years, suffering from atherosclerosis. The end of the cold to the
touch, pale, bluish tint of the foot and lower leg. Ripple on the right femoral
artery is absent.
What is the preliminary diagnosis?
A. Sharp thromboangiitis right foot
B. Deep vein thrombosis sharp right foot
C. Sharp right femoral artery spasm
D. Sharp thrombosis of the right femoral artery
E. Sharp lymphocytosis, lymphedema
14
Theme: The arterial thrombosis and embolisms
Tests (Step-2)
Variant 3
1. The patient, 76 years old, ambulance transported in serious condition with
complaints of sharp pain in the left lower limb, which appeared within hours
and growing. When walking feels the lack of solid support, cold feet. He
suffers from coronary artery disease, diabetes mellitus. On examination, the
left lower limb pale knee, shin and foot cold under the skin traced small
vessels. Pain sensitivity broken leg and foot on the left. Ripple on foot and
popliteal fossa missing left, weakened femoral arteries.
A. Popliteal artery thrombosis
B. Diabetic gangrene of the left lower limb
C. Atherosclerosis
D. Illness by Reyno
E. Thrombophlebitis
2. In the department of vascular surgery delivered from the central district
hospital patient, 66 years old, with complaints of pain, loss of sensitivity and
movement in his right foot. These symptoms arose three days ago, was
treated conservatively. Objective: body temperature - 38.2 degrees, skin
pale, blood pressure - 110/70 mm of mercury column, pulse - 110 min.,
Locally - foot and leg cold, active movement in the knee and foot are absent,
swelling and soreness of muscles legs, heart rate is determined only on the
femoral artery. What should to be the surgeon?
A. Conduct of the popliteal artery thrombectomy
B. Make fastsiotomiya on the shin
C. Hold thrombolytic therapy with streptokinase
D. Do amputation at the hip
E. Do amputation at the tibia
3. The man complained of pain in the left arm, pain in bending the elbow and
Acute radiation-carpal bones joints. Pain appeared 12 hours ago after heavy
exercise. Objective: edema of the left hand, forearm, shoulder. Skin on the
left limb cyanotic, reduced pain and tactile sensitivity. Pulse on the hand,
forearm and shoulder is not defined. Formulate a preliminary diagnosis:
15
A. Sharp thrombosis of the subclavian vein
B. Sharp brachial artery thrombosis
C. Syndrome by Reyno
D. Illness by Reyno
E. The gap groin artery
4. The patient was hospitalized in the department of vascular surgery on the
third day of onset with severe muscular contracture of the right leg. Leg
limited in all joints mobile, bluish skin color, reaction to pain stimulation
there. What is type of surgery need to do?
A. Embolectomy
B. Tromboektomiya
C. Amputation
D. Shunting
E. Prosthesis
5. In the district hospital which is located 30 km from the central district
hospital and 180 km from the regional hospital, hospitalized patient is 62
years of intramural myocardial infarction. After 25 days of onset of night he
suddenly appeared sharp pain in the left lower limb. Ending immediately
became pale. Gone active movement of toes and ankle-stage joint, decreased
skin sensitivity to palpation of the calf muscle moderately painful. Ripple in
the hip during inguinal artery connection, left - enhanced, compared with
femoral artery pulsation right. What is the most likely diagnosis?
A. Thrombosis of the femoral artery
B. Atherosclerosis of the arteries of the lower extremities
C. Embolism femoral artery
D. Illness by Reyno
E. Endarteritis obliterans of the lower extremities
16
Theme: The arterial thrombosis and embolisms
Tests (Step-2)
Variant 4
1. The patient, 52 years old, complained of sharp pain in his left leg and foot,
which came 2 hours ago when you change position in bed? Sick mitral heart
defect and atrial fibrillation. Locally: left foot and lower leg pale, cold to the
touch, active movement in the foot limited tactile sensitivity of the limbs is
reduced. Ripple in the popliteal artery and below the missing on both
femoral and popliteal right - preserved. What is causes this pathology
clinical picture?
A. Sharp thrombosis of the left femoral artery
B. Spasm of artery left lower limb
C. Atherosclerosis left lower limb
D. Left popliteal artery embolism
E. Lymphocytosis left lower limb
2. In patients suffering from heart defect and atrial fibrillation, sudden sharp
pain in the left lower leg and foot. Foot and lower third of the tibia pale, cold
to the touch. Palpation shin painful tactile sensitivity is reduced, limited
movement, pulsation of the arteries of the foot is not defined. What is the
most likely diagnosis?
A. Sharp thrombosis
B. Sharp phlebitis
C. Erysipelas
D. Thrombosis of the femoral artery
E. Gangrene
3. The patient is taken to hospital after 1 hour after the onset of severe pain in
his left leg. In history - heart defect. The left foot is cold to the touch, pale
thigh sharply, palpation patient complains of sharp pain. Ripple femoral
artery during under Pouparts ligament expressive; in other arteries of the
legs - no pulsation. Formulate a preliminary diagnosis:
A. Sharp thrombosis left leg
B. Deep vein thrombosis, sharp left leg
C. Sharp thrombosis of the left femoral artery
D. Sharp lymphocytosis, lymphedema left
17
E. Sharp left shin phlebothrombosis
4. The patient, 42 years old, who suffers from rheumatism and combined mitral
heart defect suddenly appeared sharp pain in the right upper extremity for 2
hours. In vascular angiographic examination revealed a clear contrast level
of breakage of the right brachial artery collaterals available. What is the
status of the patient?
A. Brachial artery thrombosis
B. Spasm of the brachial artery
C. Brachial artery embolism
D. Illness by Reyno
5. The patient, 35 years old, from rheumatism in the active phase has mitral
valve stenosis fourth stage, atrial fibrillation. Six days ago there were severe
pain at rest in the right lower limb, with no objective review of arterial
pulsation of the femoral artery right, foot skin temperature is lowered, bluish
color, foot movement is limited, sensitivity is preserved. Identify disease.
A. Sharp cerebrovascular accident
B. Sharp right femoral artery thromboembolism
C. Sharp deep vein thrombosis right leg
D. Erysipelas right tibia
E. Lymphangoitis sharp right lower limb
18
Theme: The arterial thrombosis and embolisms
Tests (Step-2)
Variant 5
1. The patient, 32 years old, who suffers congenital heart defect and atrial
fibrillation, sudden sharp pain in the left lower leg and foot. The patient was
examined in an hour at home. She moans in pain, foot and lower third of the
tibia pale, cold. Palpation sharply painful shin mobility in the ankle joint is
absent, superficial and deep sensitivity on the feet are missing. Pulse on
femoral arteries clear, the popliteal artery and arteries of the foot - no. What
did disease appeared in the patient?
A. Left femoral artery thrombosis
B. Sharp left popliteal artery embolism. Ischemia II B degrees. Sharp
deep vein thrombosis of the left lower limb
C. Aftertromboflebitychnyy left lower limb syndrome
D. Atherosclerosis of the left lower limb. Occlusion of the femoral-
segment hamstring
2. The patient was hospitalized in the department of vascular surgery three
days from the onset of severe muscular contracture of the right leg. Leg
limited in all joints mobile, bluish skin color, reaction to pain stimulation
there.
What is a preliminary diagnosis?
A. Sharp arterial thrombosis
B. Sharp phlebothrombosis
C. Contracture by Dupuytren
D. Sharp thrombosis
E. Position syndrome
3. The patient, 35 years old, from rheumatism in the active phase has mitral
valve stenosis fourth stage, atrial fibrillation. Six hours ago the expressed
pains appeared at a calmness in a right lower limb, at an objective review the
arterial pulsation of femoral artery is absent on the right, the temperature of
skin of foot is mionectic, bluish color, motion limits in a foot, a sensitiveness
is stored.
19
A. ECG
B. Ultrasound of the heart
C. Extremity vascular ultrasound
D. Monitoring by Holter
E. Phlebography
4. In patients with acute ischemia of the extremities of both encountered while,
but has more significant manifestations of matter, there is no ripple in the
right femoral, popliteal arteries and the arteries of the foot. Determined
relaxation ripple left thigh arteries and other arteries of the left lower limb.
Determine the level of occlusion:
F. Occlusion of the right iliac artery
G. Tromb - "rider" aortic bifurcation
H. Occlusion of both femoral arteries
I. Forensic thrombosis aortic bifurcation
J. Occlusion of the right external iliac and femoral arteries of the left
internal
5. The boy, 10 years old, incised wound of the anterior forearm. With wounds
stems thin trickle of dark blood. What method should temporarily stop
bleeding apply?
A. Finger pressing brachial artery
B. Apply braid above the wound
C. Apply bandages shaking
D. Apply braid wound below
E. Use the method to overbend limbs
20
Theme: The arterial thrombosis and embolisms
Tests (Step-2)
Variant 6
1. The patient, 62 years old, complaining of intense pain in the left leg, there
were three hours ago suddenly, numbness and cold extremities. During the
year, noted the pain in the leg while walking, increased sensitivity to
cooling. The left foot and lower leg with marble pattern, saphenas slept. At
the touch of foot cold reduced active movement of the foot, toes saved.
Pulse detectable only in the femoral artery. Above it rough systolic murmur
is heard. Formulate a preliminary diagnosis:
A. Endarteritis obliterans
B. Stenosis of the left popliteal artery
C. Sharp occlusion of the left femoral artery
D. Sharp thrombophlebitis
E. Sharp ileofemoralnyy arterial thrombosis
2. A patient suffering from combined mitral defect for 15 years. 6 hours ago
there were severe pain in the left leg. On examination, the left foot is cold to
the touch, pale from the foot to the thigh, painful on palpation, pulsation of
the femoral artery is determined only by Pouparts connection, other arteries
no pulse. What is a preliminary diagnosis?
A. Sharp thrombosis of the left femoral artery
B. Sharp trombovlebit superficial veins of the left thigh
C. Deep vein thrombosis, sharp left hip
D. Sharp lymph venous stasis left foot
E. Aneurysm of the left femoral artery
3. Patient 5 hours ago was in a car accident. He complains of abdominal pain,
headache. Blood pressure was 180/100 mmHg. In Plain radiography free
abdominal organs no fluid in the abdominal flank were found. Severe
oliguria, urine output of 10 mL / hour. In the abdomen on the white line,
largely left palpable formation 10h8 cm., tautoelastic consistency over
which auscultation systolic murmur is heard. Over the last 2:00 noticeable
trend towards larger entities. Symptoms of peritoneal irritation there. Put the
preliminary diagnosis:
21
A. Traumatic aneurysm of the abdominal aorta
B. Two-step subscapular spleen rupture
C. Retroperitoneal hematoma
D. Rupture of liver subscapular hematoma
E. Post-traumatic hematoma ripples small intestine
4. In patients with acute ischemia of the extremities of both encountered while,
but has more significant manifestations of matter, there is no ripple in the
right femoral, popliteal arteries and the arteries of the foot. Determined
relaxation ripple left thigh arteries and other arteries of the left lower limb.
What is a complementary method to assign?
A. Vascular ultrasound
B. Angiography of aorta
C. Phlebography
D. Ultrasound of the heart
5. For a patient with the sharp ischemia of both lower limbs, that arose up
simultaneously, absent pulsation on right and left femoral, popliteal arteries.
Great pain appeared in a stomach, vomit, collapse, liquid chairs, positive
symptoms of irritation of peritoneum. What is the condition of the patient?
A. Occlusion of right iliac artery
B. Thrombosis and embolism aortic bifurcation inferior mesenteric artery
C. Thrombosis of both femoral arteries
D. Layering aortic aneurysm
E. Total thrombosis of bifurcation of aorta
22
Theme: The arterial thrombosis and embolisms
Tests (Step-2)
Variant 7
1. The patient came to the clinic 3 hours ago with the onset of pain in his right
leg. Sick 10 years, suffering from atherosclerosis. The end of the cold to the
touch, pale, bluish tint of the foot and lower leg. Ripple on the right femoral
artery is absent.
What is the preliminary diagnosis?
A. Sharp thromboangiitis right foot
B. Deep vein thrombosis sharp right foot
C. Sharp right femoral artery spasm
D. Sharp thrombosis of the right femoral artery
E. Sharp lymphocytosis, lymphedema
2. The patient, 45 years old, is on treatment for rheumatism, active phase,
combined mitral valve defect. During the morning the toilet suddenly felt
pain in his left hand, followed her numbness. Pain and numbness are grew.
Objective: pale skin of his left hand, relatively cold. Pulsation of the arteries
throughout the hand is missing.
What is the most appropriate therapeutic approach?
A. Appointment fibrinolysis, anticoagulants
B. Purpose of antibiotics and anti-inflammatory drugs
C. Emergency embolectomy
D. Probing heart
E. Emergency trombintymektomiya
3. The boy, 10 years old, incised wound of the anterior forearm. With wounds
stems thin trickle of dark blood. What method should temporarily stop
bleeding apply?
A. Finger pressing brachial artery
B. Apply braid above the wound
C. Apply bandages shaking
D. Apply braid wound below
E. Use the method to overbend limbs
23
4. The patient, 32 years old, who suffers congenital heart defect and atrial
fibrillation, sudden sharp pain in the left lower leg and foot. The patient was
examined in an hour at home. She moans in pain, foot and lower third of the
tibia pale, cold. Palpation sharply painful shin mobility in the ankle joint is
absent, superficial and deep sensitivity on the feet are missing. Pulse on
femoral arteries clear, the popliteal artery and arteries of the foot - no. What
did disease appeared in the patient?
A. Left femoral artery thrombosis
B. Sharp left popliteal artery embolism. Ischemia II B degrees. Sharp
deep vein thrombosis of the left lower limb
C. Aftertromboflebitychnyy left lower limb syndrome
D. Atherosclerosis of the left lower limb. Occlusion of the femoral-
segment hamstring.
5. In the department of vascular surgery delivered from the central district
hospital patient, 66 years old, with complaints of pain, loss of sensitivity and
movement in his right foot. These symptoms arose three days ago, was
treated conservatively. Objective: body temperature - 38.2 degrees, skin
pale, blood pressure - 110/70 mm of mercury column, pulse - 110 min.,
Locally - foot and leg cold, active movement in the knee and foot are absent,
swelling and soreness of muscles legs, heart rate is determined only on the
femoral artery. What should to be the surgeon?
A. Conduct of the popliteal artery thrombectomy
B. Make fastsiotomiya on the shin
C. Hold thrombolytic therapy with streptokinase
D. Do amputation at the hip
E. Do amputation at the tibia
24
Theme: The arterial thrombosis and embolisms
Tests (Step-2)
Variant 8
1. A patient with the sharp ischemia of both lower limbs, that arose up
simultaneously, absent pulsation on right and left femoral, popliteal arteries.
Great pain appeared in a stomach, vomit, collapse, liquid chairs, positive
symptoms of irritation of peritoneum. What is the condition of the patient?
A. Occlusion of right iliac artery
B. Thrombosis and embolism aortic bifurcation inferior mesenteric artery
C. Thrombosis of both femoral arteries
D. Layering aortic aneurysm
E. Total thrombosis of bifurcation of aorta
2. The man complained of pain in the left arm, pain in bending the elbow and
Acute radiation-carpal bones joints. Pain appeared 12 hours ago after heavy
exercise. Objective: edema of the left hand, forearm, shoulder. Skin on the
left limb cyanotic, reduced pain and tactile sensitivity. Pulse on the hand,
forearm and shoulder is not defined. Formulate a preliminary diagnosis:
A. Sharp thrombosis of the subclavian vein
B. Sharp brachial artery thrombosis
C. Syndrome by Reyno
D. Illness by Reyno
E. The gap groin artery
3. In the district hospital which is located 30 km from the central district
hospital and 180 km from the regional hospital, hospitalized patient is 62
years of intramural myocardial infarction. After 25 days of onset of night he
suddenly appeared sharp pain in the left lower limb. Ending immediately
became pale. Gone active movement of toes and ankle-stage joint, decreased
skin sensitivity to palpation of the calf muscle moderately painful. Ripple in
the hip during inguinal artery connection, left - enhanced, compared with
femoral artery pulsation right. What is the most likely diagnosis?
A. Thrombosis of the femoral artery
B. Atherosclerosis of the arteries of the lower extremities
C. Embolism femoral artery
D. Illness by Reyno
25
E. Endarteritis obliterans of the lower extremities
4. The patient, 62 years old, complaining of intense pain in the left leg, there
were three hours ago suddenly, numbness and cold extremities. During the
year, noted the pain in the leg while walking, increased sensitivity to
cooling. The left foot and lower leg with marble pattern, saphenas slept. At
the touch of foot cold reduced active movement of the foot, toes saved.
Pulse detectable only in the femoral artery. Above it rough systolic murmur
is heard. Formulate a preliminary diagnosis:
A. Endarteritis obliterans
B. Stenosis of the left popliteal artery
C. Sharp occlusion of the left femoral artery
D. Sharp thrombophlebitis
E. Sharp ileofemoralnyy
5. The patient, 76 years old, ambulance transported in serious condition with
complaints of sharp pain in the left lower limb, which appeared within hours
and growing. When walking feels the lack of solid support, cold feet. He
suffers from coronary artery disease, diabetes mellitus. On examination, the
left lower limb pale knee, shin and foot cold under the skin traced small
vessels. Pain sensitivity broken leg and foot on the left. Ripple on foot and
popliteal fossa missing left, weakened femoral arteries.
A. Popliteal artery thrombosis
B. Diabetic gangrene of the left lower limb
C. Atherosclerosis
D. Illness by Reyno
E. Thrombophlebitis
26
Theme: The arterial thrombosis and embolisms
Tests (Step-2)
Variant 9
1. The boy, 10 years old, incised wound of the anterior forearm. With wounds
stems thin trickle of dark blood. What method should temporarily stop
bleeding apply?
A. Finger pressing brachial artery
B. Apply braid above the wound
C. Apply bandages shaking
D. Apply braid wound below
E. Use the method to overbend limbs
2. The patient was hospitalized in the department of vascular surgery on the
third day of onset with severe muscular contracture of the right leg. Leg
limited in all joints mobile, bluish skin color, reaction to pain stimulation
there. What is type of surgery need to do?
A. Embolectomy
B. Tromboektomiya
C. Amputation
D. Shunting
E. Prosthesis
3. In patients suffering from heart defect and atrial fibrillation, sudden sharp
pain in the left lower leg and foot. Foot and lower third of the tibia pale, cold
to the touch. Palpation shin painful tactile sensitivity is reduced, limited
movement, pulsation of the arteries of the foot is not defined. What is the
most likely diagnosis?
A. Sharp thrombosis
B. Sharp phlebitis
C. Erysipelas
D. Thrombosis of the femoral artery
E. Gangrene
4. The patient, 42 years old, who suffers from rheumatism and combined mitral
heart defect suddenly appeared sharp pain in the right upper extremity for 2
hours. In vascular angiographic examination revealed a clear contrast level
27
of breakage of the right brachial artery collaterals available. What is the
status of the patient?
A. Brachial artery thrombosis
B. Spasm of the brachial artery
C. Brachial artery embolism
D. Illness by Reyno
5. In the department of vascular surgery delivered from the central district
hospital patient, 66 years old, with complaints of pain, loss of sensitivity and
movement in his right foot. These symptoms arose three days ago, was
treated conservatively. Objective: body temperature - 38.2 degrees, skin
pale, blood pressure - 110/70 mm of mercury column, pulse - 110 min.,
Locally - foot and leg cold, active movement in the knee and foot are absent,
swelling and soreness of muscles legs, heart rate is determined only on the
femoral artery. What should to be the surgeon?
A. Conduct of the popliteal artery thrombectomy
B. Make fastsiotomiya on the shin
C. Hold thrombolytic therapy with streptokinase
D. Do amputation at the hip
E. Do amputation at the tibia
28
Theme: The arterial thrombosis and embolisms
Tests (Step-2)
Variant 10
1. The patient, 60 years old, there was a sharp pain in the left lower limb,
which appeared within hours and growing. Vascular Angiography revealed
that the contours of the popliteal artery uneven, irregular line break, plenty
of collaterals. What is the status of the patient?
A. Popliteal artery embolism
B. Trumbo popliteal artery
C. Spasm of the popliteal artery
D. Illness by Reyno
E. Tthrombophlebitis
2. The patient, 62 years old, complaining of intense pain in the left leg, there
were three hours ago suddenly, numbness and cold extremities. During the
year, noted the pain in the leg while walking, increased sensitivity to
cooling. The left foot and lower leg with marble pattern, saphenas slept. At
the touch of foot cold reduced active movement of the foot, toes saved.
Pulse detectable only in the femoral artery. Above it rough systolic murmur
is heard. Formulate a preliminary diagnosis:
A. Endarteritis obliterans
B. Stenosis of the left popliteal artery
C. Sharp occlusion of the left femoral artery
D. Sharp thrombophlebitis
E. Sharp ileofemoralnyy arterial thrombosis
3. The patient, 56 years old, complaining of intense pain in the left leg, there
were three hours ago suddenly, numbness and cold extremities. During the
year, noted the pain in the leg while walking, increased sensitivity to
cooling. The left foot and lower leg with marble pattern, marked "a
symptom of the groove." At the touch of foot cold reduced active movement
of the foot, toes saved. Pulse detectable only in the femoral artery. Above
her systolic murmur is heard. Formulate a preliminary diagnosis:
A. Endarteritis obliterans
B. Syndrome by Reyno
C. Sharp thrombophlebitis
29
D. Atherosclerosis of vessels of lower limbs, sharp thrombosis
E. Illness by Buerger
4. In patients with acute ischemia of the extremities of both encountered while,
but has more significant manifestations of matter, there is no ripple in the
right femoral, popliteal arteries and the arteries of the foot. Determined
relaxation ripple left thigh arteries and other arteries of the left lower limb.
What is a complementary method to assign?
A. Vascular ultrasound
B. Angiography of aorta
C. Phlebography
D. Ultrasound of the heart
5. The patient, 32 years old, who suffers congenital heart defect and atrial
fibrillation, sudden sharp pain in the left lower leg and foot. The patient was
examined in an hour at home. She moans in pain, foot and lower third of the
tibia pale, cold. Palpation sharply painful shin mobility in the ankle joint is
absent, superficial and deep sensitivity on the feet are missing. Pulse on
femoral arteries clear, the popliteal artery and arteries of the foot - no. What
did disease appeared in the patient?
A. Left femoral artery thrombosis
B. Sharp left popliteal artery embolism. Ischemia II B degrees. Sharp
deep vein thrombosis of the left lower limb
C. Aftertromboflebitychnyy left lower limb syndrome
D. Atherosclerosis of the left lower limb. Occlusion of the femoral-
segment hamstring
30
Theme: The arterial thrombosis and embolisms
Tests (Step-2)
Variant 11
1. The patient, 52 years old, complained of sharp pain in his left leg and foot,
which came 2 hours ago when you change position in bed? Sick mitral heart
defect and atrial fibrillation. Locally: left foot and lower leg pale, cold to the
touch, active movement in the foot limited tactile sensitivity of the limbs is
reduced. Ripple in the popliteal artery and below the missing on both
femoral and popliteal right - preserved. What is causes this pathology
clinical picture?
A. Sharp thrombosis of the left femoral artery
B. Spasm of artery left lower limb
C. Atherosclerosis left lower limb
D. Left popliteal artery embolism
E. Lymphocytosis left lower limb
2. The patient is taken to hospital after 1 hour after the onset of severe pain in
his left leg. In history - heart defect. The left foot is cold to the touch, pale
thigh sharply, palpation patient complains of sharp pain. Ripple femoral
artery during under Pouparts ligament expressive; in other arteries of the
legs - no pulsation. Formulate a preliminary diagnosis:
A. Sharp thrombosis left leg
B. Deep vein thrombosis, sharp left leg
C. Sharp thrombosis of the left femoral artery
D. Sharp lymphocytosis, lymphedema left
E. Sharp left shin phlebothrombosis
3. The patient, 76 years old, ambulance transported in serious condition with
complaints of sharp pain in the left lower limb, which appeared within hours
and growing. When walking feels the lack of solid support, cold feet. He
suffers from coronary artery disease, diabetes mellitus. On examination, the
left lower limb pale knee, shin and foot cold under the skin traced small
vessels. Pain sensitivity broken leg and foot on the left. Ripple on foot and
popliteal fossa missing left, weakened femoral arteries.
A. Popliteal artery thrombosis
B. Diabetic gangrene of the left lower limb
31
C. Atherosclerosis
D. Illness by Reyno
E. Thrombophlebitis
4. The patient was hospitalized in the department of vascular surgery three
days from the onset of severe muscular contracture of the right leg. Leg
limited in all joints mobile, bluish skin color, reaction to pain stimulation
there.
What is a preliminary diagnosis?
A. Sharp arterial thrombosis
B. Sharp phlebothrombosis
C. Contracture by Dupuytren
D. Sharp thrombosis
E. Position syndrome
5. The patient, 35 years old, from rheumatism in the active phase has mitral
valve stenosis fourth stage, atrial fibrillation. Six days ago there were severe
pain at rest in the right lower limb, with no objective review of arterial
pulsation of the femoral artery right, foot skin temperature is lowered, bluish
color, foot movement is limited, sensitivity is preserved. Identify disease.
A. Sharp cerebrovascular accident
B. Sharp right femoral artery thromboembolism
C. Sharp deep vein thrombosis right leg
D. Erysipelas right tibia
E. Lymphangoitis sharp right lower limb

More Related Content

Similar to Arterial_thrombosis_and_embolism-1.pdf

11. vascular disease
11. vascular disease11. vascular disease
11. vascular diseaseAhmad Hamadi
 
Cardiovascular Pathophysiology- Coronary Artery Disease (CAD)/Ischaemic Heart...
Cardiovascular Pathophysiology- Coronary Artery Disease (CAD)/Ischaemic Heart...Cardiovascular Pathophysiology- Coronary Artery Disease (CAD)/Ischaemic Heart...
Cardiovascular Pathophysiology- Coronary Artery Disease (CAD)/Ischaemic Heart...VISHALJADHAV100
 
CVS pathology 3 - ischemic heart disease, angina and myocardial infarction 20...
CVS pathology 3 - ischemic heart disease, angina and myocardial infarction 20...CVS pathology 3 - ischemic heart disease, angina and myocardial infarction 20...
CVS pathology 3 - ischemic heart disease, angina and myocardial infarction 20...Sufia Husain
 
Management of aorto arteritis
Management of aorto arteritisManagement of aorto arteritis
Management of aorto arteritisIndia CTVS
 
Lower limb ischemia
Lower limb ischemiaLower limb ischemia
Lower limb ischemiaJohn Mathew
 
strokeppt-170720174010.pdf
strokeppt-170720174010.pdfstrokeppt-170720174010.pdf
strokeppt-170720174010.pdfRiyaSharma295
 
ISCHEMIC HEART DISEASE (IHD)
ISCHEMIC HEART DISEASE (IHD)ISCHEMIC HEART DISEASE (IHD)
ISCHEMIC HEART DISEASE (IHD)SAMOEINESH
 
Case record...Cervical vascular spondylotic myelopathy
Case record...Cervical vascular spondylotic myelopathyCase record...Cervical vascular spondylotic myelopathy
Case record...Cervical vascular spondylotic myelopathyProfessor Yasser Metwally
 
upper limb ischemia ppt.pptx
upper limb ischemia ppt.pptxupper limb ischemia ppt.pptx
upper limb ischemia ppt.pptxPRAGATISHUKLA40
 
Myocardial infarction
Myocardial infarction Myocardial infarction
Myocardial infarction vani
 
Peripheral artery disease
Peripheral artery diseasePeripheral artery disease
Peripheral artery diseasemahboobehsheikh
 
ARTERIOSCLEROSIS.pptx
ARTERIOSCLEROSIS.pptxARTERIOSCLEROSIS.pptx
ARTERIOSCLEROSIS.pptxSAMOEINESH
 

Similar to Arterial_thrombosis_and_embolism-1.pdf (20)

Jcp05500481 3
Jcp05500481 3Jcp05500481 3
Jcp05500481 3
 
stroke
 stroke stroke
stroke
 
Acute Limb Ischemia
Acute Limb IschemiaAcute Limb Ischemia
Acute Limb Ischemia
 
11. vascular disease
11. vascular disease11. vascular disease
11. vascular disease
 
Cardiovascular Pathophysiology- Coronary Artery Disease (CAD)/Ischaemic Heart...
Cardiovascular Pathophysiology- Coronary Artery Disease (CAD)/Ischaemic Heart...Cardiovascular Pathophysiology- Coronary Artery Disease (CAD)/Ischaemic Heart...
Cardiovascular Pathophysiology- Coronary Artery Disease (CAD)/Ischaemic Heart...
 
CVS pathology 3 - ischemic heart disease, angina and myocardial infarction 20...
CVS pathology 3 - ischemic heart disease, angina and myocardial infarction 20...CVS pathology 3 - ischemic heart disease, angina and myocardial infarction 20...
CVS pathology 3 - ischemic heart disease, angina and myocardial infarction 20...
 
Management of aorto arteritis
Management of aorto arteritisManagement of aorto arteritis
Management of aorto arteritis
 
Lower limb ischemia
Lower limb ischemiaLower limb ischemia
Lower limb ischemia
 
strokeppt-170720174010.pdf
strokeppt-170720174010.pdfstrokeppt-170720174010.pdf
strokeppt-170720174010.pdf
 
Stroke ppt
Stroke pptStroke ppt
Stroke ppt
 
ISCHEMIC HEART DISEASE (IHD)
ISCHEMIC HEART DISEASE (IHD)ISCHEMIC HEART DISEASE (IHD)
ISCHEMIC HEART DISEASE (IHD)
 
Case record...Cervical vascular spondylotic myelopathy
Case record...Cervical vascular spondylotic myelopathyCase record...Cervical vascular spondylotic myelopathy
Case record...Cervical vascular spondylotic myelopathy
 
upper limb ischemia ppt.pptx
upper limb ischemia ppt.pptxupper limb ischemia ppt.pptx
upper limb ischemia ppt.pptx
 
Myocardial infarction
Myocardial infarction Myocardial infarction
Myocardial infarction
 
Acute limb ischemia
Acute limb ischemiaAcute limb ischemia
Acute limb ischemia
 
ali.pdf
ali.pdfali.pdf
ali.pdf
 
Peripheral artery disease
Peripheral artery diseasePeripheral artery disease
Peripheral artery disease
 
Atherosclerosis
AtherosclerosisAtherosclerosis
Atherosclerosis
 
ARTERIOSCLEROSIS.pptx
ARTERIOSCLEROSIS.pptxARTERIOSCLEROSIS.pptx
ARTERIOSCLEROSIS.pptx
 
CVD.pptx
CVD.pptxCVD.pptx
CVD.pptx
 

More from ssuser886f40

Epidemiology and prophylaxis of helminthiasis Amina.ppt
Epidemiology and prophylaxis of helminthiasis Amina.pptEpidemiology and prophylaxis of helminthiasis Amina.ppt
Epidemiology and prophylaxis of helminthiasis Amina.pptssuser886f40
 
6.1.THYROIDITIS.ppt
6.1.THYROIDITIS.ppt6.1.THYROIDITIS.ppt
6.1.THYROIDITIS.pptssuser886f40
 
immunization forvarious diseases.pptx
immunization forvarious diseases.pptximmunization forvarious diseases.pptx
immunization forvarious diseases.pptxssuser886f40
 
экстренные состояния в паллиативной помощи.ppt
экстренные состояния в паллиативной помощи.pptэкстренные состояния в паллиативной помощи.ppt
экстренные состояния в паллиативной помощи.pptssuser886f40
 
Элективный курс Хирурии.ppt
Элективный курс Хирурии.pptЭлективный курс Хирурии.ppt
Элективный курс Хирурии.pptssuser886f40
 
Элективы.ppt
Элективы.pptЭлективы.ppt
Элективы.pptssuser886f40
 
Доклад тип 1.ppt
Доклад тип 1.pptДоклад тип 1.ppt
Доклад тип 1.pptssuser886f40
 
Презентация1.ppt
Презентация1.pptПрезентация1.ppt
Презентация1.pptssuser886f40
 
SC_S2_Thyroid_and_Parathyroid_Disease.ppt
SC_S2_Thyroid_and_Parathyroid_Disease.pptSC_S2_Thyroid_and_Parathyroid_Disease.ppt
SC_S2_Thyroid_and_Parathyroid_Disease.pptssuser886f40
 

More from ssuser886f40 (10)

Epidemiology and prophylaxis of helminthiasis Amina.ppt
Epidemiology and prophylaxis of helminthiasis Amina.pptEpidemiology and prophylaxis of helminthiasis Amina.ppt
Epidemiology and prophylaxis of helminthiasis Amina.ppt
 
6.1.THYROIDITIS.ppt
6.1.THYROIDITIS.ppt6.1.THYROIDITIS.ppt
6.1.THYROIDITIS.ppt
 
immunization forvarious diseases.pptx
immunization forvarious diseases.pptximmunization forvarious diseases.pptx
immunization forvarious diseases.pptx
 
экстренные состояния в паллиативной помощи.ppt
экстренные состояния в паллиативной помощи.pptэкстренные состояния в паллиативной помощи.ppt
экстренные состояния в паллиативной помощи.ppt
 
Элективный курс Хирурии.ppt
Элективный курс Хирурии.pptЭлективный курс Хирурии.ppt
Элективный курс Хирурии.ppt
 
Элективы.ppt
Элективы.pptЭлективы.ppt
Элективы.ppt
 
Доклад тип 1.ppt
Доклад тип 1.pptДоклад тип 1.ppt
Доклад тип 1.ppt
 
Презентация1.ppt
Презентация1.pptПрезентация1.ppt
Презентация1.ppt
 
SC_S2_Thyroid_and_Parathyroid_Disease.ppt
SC_S2_Thyroid_and_Parathyroid_Disease.pptSC_S2_Thyroid_and_Parathyroid_Disease.ppt
SC_S2_Thyroid_and_Parathyroid_Disease.ppt
 
Open damage.ppt
Open damage.pptOpen damage.ppt
Open damage.ppt
 

Recently uploaded

💎VVIP Kolkata Call Girls Parganas🩱7001035870🩱Independent Girl ( Ac Rooms Avai...
💎VVIP Kolkata Call Girls Parganas🩱7001035870🩱Independent Girl ( Ac Rooms Avai...💎VVIP Kolkata Call Girls Parganas🩱7001035870🩱Independent Girl ( Ac Rooms Avai...
💎VVIP Kolkata Call Girls Parganas🩱7001035870🩱Independent Girl ( Ac Rooms Avai...Taniya Sharma
 
The Most Attractive Hyderabad Call Girls Kothapet 𖠋 6297143586 𖠋 Will You Mis...
The Most Attractive Hyderabad Call Girls Kothapet 𖠋 6297143586 𖠋 Will You Mis...The Most Attractive Hyderabad Call Girls Kothapet 𖠋 6297143586 𖠋 Will You Mis...
The Most Attractive Hyderabad Call Girls Kothapet 𖠋 6297143586 𖠋 Will You Mis...chandars293
 
VIP Service Call Girls Sindhi Colony 📳 7877925207 For 18+ VIP Call Girl At Th...
VIP Service Call Girls Sindhi Colony 📳 7877925207 For 18+ VIP Call Girl At Th...VIP Service Call Girls Sindhi Colony 📳 7877925207 For 18+ VIP Call Girl At Th...
VIP Service Call Girls Sindhi Colony 📳 7877925207 For 18+ VIP Call Girl At Th...jageshsingh5554
 
Chandrapur Call girls 8617370543 Provides all area service COD available
Chandrapur Call girls 8617370543 Provides all area service COD availableChandrapur Call girls 8617370543 Provides all area service COD available
Chandrapur Call girls 8617370543 Provides all area service COD availableDipal Arora
 
Call Girl Number in Vashi Mumbai📲 9833363713 💞 Full Night Enjoy
Call Girl Number in Vashi Mumbai📲 9833363713 💞 Full Night EnjoyCall Girl Number in Vashi Mumbai📲 9833363713 💞 Full Night Enjoy
Call Girl Number in Vashi Mumbai📲 9833363713 💞 Full Night Enjoybabeytanya
 
Kesar Bagh Call Girl Price 9548273370 , Lucknow Call Girls Service
Kesar Bagh Call Girl Price 9548273370 , Lucknow Call Girls ServiceKesar Bagh Call Girl Price 9548273370 , Lucknow Call Girls Service
Kesar Bagh Call Girl Price 9548273370 , Lucknow Call Girls Servicemakika9823
 
Lucknow Call girls - 8800925952 - 24x7 service with hotel room
Lucknow Call girls - 8800925952 - 24x7 service with hotel roomLucknow Call girls - 8800925952 - 24x7 service with hotel room
Lucknow Call girls - 8800925952 - 24x7 service with hotel roomdiscovermytutordmt
 
Call Girls Bhubaneswar Just Call 9907093804 Top Class Call Girl Service Avail...
Call Girls Bhubaneswar Just Call 9907093804 Top Class Call Girl Service Avail...Call Girls Bhubaneswar Just Call 9907093804 Top Class Call Girl Service Avail...
Call Girls Bhubaneswar Just Call 9907093804 Top Class Call Girl Service Avail...Dipal Arora
 
Bangalore Call Girls Nelamangala Number 7001035870 Meetin With Bangalore Esc...
Bangalore Call Girls Nelamangala Number 7001035870  Meetin With Bangalore Esc...Bangalore Call Girls Nelamangala Number 7001035870  Meetin With Bangalore Esc...
Bangalore Call Girls Nelamangala Number 7001035870 Meetin With Bangalore Esc...narwatsonia7
 
Call Girls Darjeeling Just Call 9907093804 Top Class Call Girl Service Available
Call Girls Darjeeling Just Call 9907093804 Top Class Call Girl Service AvailableCall Girls Darjeeling Just Call 9907093804 Top Class Call Girl Service Available
Call Girls Darjeeling Just Call 9907093804 Top Class Call Girl Service AvailableDipal Arora
 
♛VVIP Hyderabad Call Girls Chintalkunta🖕7001035870🖕Riya Kappor Top Call Girl ...
♛VVIP Hyderabad Call Girls Chintalkunta🖕7001035870🖕Riya Kappor Top Call Girl ...♛VVIP Hyderabad Call Girls Chintalkunta🖕7001035870🖕Riya Kappor Top Call Girl ...
♛VVIP Hyderabad Call Girls Chintalkunta🖕7001035870🖕Riya Kappor Top Call Girl ...astropune
 
Top Rated Bangalore Call Girls Mg Road ⟟ 8250192130 ⟟ Call Me For Genuine Sex...
Top Rated Bangalore Call Girls Mg Road ⟟ 8250192130 ⟟ Call Me For Genuine Sex...Top Rated Bangalore Call Girls Mg Road ⟟ 8250192130 ⟟ Call Me For Genuine Sex...
Top Rated Bangalore Call Girls Mg Road ⟟ 8250192130 ⟟ Call Me For Genuine Sex...narwatsonia7
 
Call Girl Number in Panvel Mumbai📲 9833363713 💞 Full Night Enjoy
Call Girl Number in Panvel Mumbai📲 9833363713 💞 Full Night EnjoyCall Girl Number in Panvel Mumbai📲 9833363713 💞 Full Night Enjoy
Call Girl Number in Panvel Mumbai📲 9833363713 💞 Full Night Enjoybabeytanya
 
Call Girls Colaba Mumbai ❤️ 9920874524 👈 Cash on Delivery
Call Girls Colaba Mumbai ❤️ 9920874524 👈 Cash on DeliveryCall Girls Colaba Mumbai ❤️ 9920874524 👈 Cash on Delivery
Call Girls Colaba Mumbai ❤️ 9920874524 👈 Cash on Deliverynehamumbai
 
Premium Call Girls Cottonpet Whatsapp 7001035870 Independent Escort Service
Premium Call Girls Cottonpet Whatsapp 7001035870 Independent Escort ServicePremium Call Girls Cottonpet Whatsapp 7001035870 Independent Escort Service
Premium Call Girls Cottonpet Whatsapp 7001035870 Independent Escort Servicevidya singh
 
Night 7k to 12k Navi Mumbai Call Girl Photo 👉 BOOK NOW 9833363713 👈 ♀️ night ...
Night 7k to 12k Navi Mumbai Call Girl Photo 👉 BOOK NOW 9833363713 👈 ♀️ night ...Night 7k to 12k Navi Mumbai Call Girl Photo 👉 BOOK NOW 9833363713 👈 ♀️ night ...
Night 7k to 12k Navi Mumbai Call Girl Photo 👉 BOOK NOW 9833363713 👈 ♀️ night ...aartirawatdelhi
 
Call Girl Coimbatore Prisha☎️ 8250192130 Independent Escort Service Coimbatore
Call Girl Coimbatore Prisha☎️  8250192130 Independent Escort Service CoimbatoreCall Girl Coimbatore Prisha☎️  8250192130 Independent Escort Service Coimbatore
Call Girl Coimbatore Prisha☎️ 8250192130 Independent Escort Service Coimbatorenarwatsonia7
 
(👑VVIP ISHAAN ) Russian Call Girls Service Navi Mumbai🖕9920874524🖕Independent...
(👑VVIP ISHAAN ) Russian Call Girls Service Navi Mumbai🖕9920874524🖕Independent...(👑VVIP ISHAAN ) Russian Call Girls Service Navi Mumbai🖕9920874524🖕Independent...
(👑VVIP ISHAAN ) Russian Call Girls Service Navi Mumbai🖕9920874524🖕Independent...Taniya Sharma
 
Russian Escorts Girls Nehru Place ZINATHI 🔝9711199012 ☪ 24/7 Call Girls Delhi
Russian Escorts Girls  Nehru Place ZINATHI 🔝9711199012 ☪ 24/7 Call Girls DelhiRussian Escorts Girls  Nehru Place ZINATHI 🔝9711199012 ☪ 24/7 Call Girls Delhi
Russian Escorts Girls Nehru Place ZINATHI 🔝9711199012 ☪ 24/7 Call Girls DelhiAlinaDevecerski
 

Recently uploaded (20)

Escort Service Call Girls In Sarita Vihar,, 99530°56974 Delhi NCR
Escort Service Call Girls In Sarita Vihar,, 99530°56974 Delhi NCREscort Service Call Girls In Sarita Vihar,, 99530°56974 Delhi NCR
Escort Service Call Girls In Sarita Vihar,, 99530°56974 Delhi NCR
 
💎VVIP Kolkata Call Girls Parganas🩱7001035870🩱Independent Girl ( Ac Rooms Avai...
💎VVIP Kolkata Call Girls Parganas🩱7001035870🩱Independent Girl ( Ac Rooms Avai...💎VVIP Kolkata Call Girls Parganas🩱7001035870🩱Independent Girl ( Ac Rooms Avai...
💎VVIP Kolkata Call Girls Parganas🩱7001035870🩱Independent Girl ( Ac Rooms Avai...
 
The Most Attractive Hyderabad Call Girls Kothapet 𖠋 6297143586 𖠋 Will You Mis...
The Most Attractive Hyderabad Call Girls Kothapet 𖠋 6297143586 𖠋 Will You Mis...The Most Attractive Hyderabad Call Girls Kothapet 𖠋 6297143586 𖠋 Will You Mis...
The Most Attractive Hyderabad Call Girls Kothapet 𖠋 6297143586 𖠋 Will You Mis...
 
VIP Service Call Girls Sindhi Colony 📳 7877925207 For 18+ VIP Call Girl At Th...
VIP Service Call Girls Sindhi Colony 📳 7877925207 For 18+ VIP Call Girl At Th...VIP Service Call Girls Sindhi Colony 📳 7877925207 For 18+ VIP Call Girl At Th...
VIP Service Call Girls Sindhi Colony 📳 7877925207 For 18+ VIP Call Girl At Th...
 
Chandrapur Call girls 8617370543 Provides all area service COD available
Chandrapur Call girls 8617370543 Provides all area service COD availableChandrapur Call girls 8617370543 Provides all area service COD available
Chandrapur Call girls 8617370543 Provides all area service COD available
 
Call Girl Number in Vashi Mumbai📲 9833363713 💞 Full Night Enjoy
Call Girl Number in Vashi Mumbai📲 9833363713 💞 Full Night EnjoyCall Girl Number in Vashi Mumbai📲 9833363713 💞 Full Night Enjoy
Call Girl Number in Vashi Mumbai📲 9833363713 💞 Full Night Enjoy
 
Kesar Bagh Call Girl Price 9548273370 , Lucknow Call Girls Service
Kesar Bagh Call Girl Price 9548273370 , Lucknow Call Girls ServiceKesar Bagh Call Girl Price 9548273370 , Lucknow Call Girls Service
Kesar Bagh Call Girl Price 9548273370 , Lucknow Call Girls Service
 
Lucknow Call girls - 8800925952 - 24x7 service with hotel room
Lucknow Call girls - 8800925952 - 24x7 service with hotel roomLucknow Call girls - 8800925952 - 24x7 service with hotel room
Lucknow Call girls - 8800925952 - 24x7 service with hotel room
 
Call Girls Bhubaneswar Just Call 9907093804 Top Class Call Girl Service Avail...
Call Girls Bhubaneswar Just Call 9907093804 Top Class Call Girl Service Avail...Call Girls Bhubaneswar Just Call 9907093804 Top Class Call Girl Service Avail...
Call Girls Bhubaneswar Just Call 9907093804 Top Class Call Girl Service Avail...
 
Bangalore Call Girls Nelamangala Number 7001035870 Meetin With Bangalore Esc...
Bangalore Call Girls Nelamangala Number 7001035870  Meetin With Bangalore Esc...Bangalore Call Girls Nelamangala Number 7001035870  Meetin With Bangalore Esc...
Bangalore Call Girls Nelamangala Number 7001035870 Meetin With Bangalore Esc...
 
Call Girls Darjeeling Just Call 9907093804 Top Class Call Girl Service Available
Call Girls Darjeeling Just Call 9907093804 Top Class Call Girl Service AvailableCall Girls Darjeeling Just Call 9907093804 Top Class Call Girl Service Available
Call Girls Darjeeling Just Call 9907093804 Top Class Call Girl Service Available
 
♛VVIP Hyderabad Call Girls Chintalkunta🖕7001035870🖕Riya Kappor Top Call Girl ...
♛VVIP Hyderabad Call Girls Chintalkunta🖕7001035870🖕Riya Kappor Top Call Girl ...♛VVIP Hyderabad Call Girls Chintalkunta🖕7001035870🖕Riya Kappor Top Call Girl ...
♛VVIP Hyderabad Call Girls Chintalkunta🖕7001035870🖕Riya Kappor Top Call Girl ...
 
Top Rated Bangalore Call Girls Mg Road ⟟ 8250192130 ⟟ Call Me For Genuine Sex...
Top Rated Bangalore Call Girls Mg Road ⟟ 8250192130 ⟟ Call Me For Genuine Sex...Top Rated Bangalore Call Girls Mg Road ⟟ 8250192130 ⟟ Call Me For Genuine Sex...
Top Rated Bangalore Call Girls Mg Road ⟟ 8250192130 ⟟ Call Me For Genuine Sex...
 
Call Girl Number in Panvel Mumbai📲 9833363713 💞 Full Night Enjoy
Call Girl Number in Panvel Mumbai📲 9833363713 💞 Full Night EnjoyCall Girl Number in Panvel Mumbai📲 9833363713 💞 Full Night Enjoy
Call Girl Number in Panvel Mumbai📲 9833363713 💞 Full Night Enjoy
 
Call Girls Colaba Mumbai ❤️ 9920874524 👈 Cash on Delivery
Call Girls Colaba Mumbai ❤️ 9920874524 👈 Cash on DeliveryCall Girls Colaba Mumbai ❤️ 9920874524 👈 Cash on Delivery
Call Girls Colaba Mumbai ❤️ 9920874524 👈 Cash on Delivery
 
Premium Call Girls Cottonpet Whatsapp 7001035870 Independent Escort Service
Premium Call Girls Cottonpet Whatsapp 7001035870 Independent Escort ServicePremium Call Girls Cottonpet Whatsapp 7001035870 Independent Escort Service
Premium Call Girls Cottonpet Whatsapp 7001035870 Independent Escort Service
 
Night 7k to 12k Navi Mumbai Call Girl Photo 👉 BOOK NOW 9833363713 👈 ♀️ night ...
Night 7k to 12k Navi Mumbai Call Girl Photo 👉 BOOK NOW 9833363713 👈 ♀️ night ...Night 7k to 12k Navi Mumbai Call Girl Photo 👉 BOOK NOW 9833363713 👈 ♀️ night ...
Night 7k to 12k Navi Mumbai Call Girl Photo 👉 BOOK NOW 9833363713 👈 ♀️ night ...
 
Call Girl Coimbatore Prisha☎️ 8250192130 Independent Escort Service Coimbatore
Call Girl Coimbatore Prisha☎️  8250192130 Independent Escort Service CoimbatoreCall Girl Coimbatore Prisha☎️  8250192130 Independent Escort Service Coimbatore
Call Girl Coimbatore Prisha☎️ 8250192130 Independent Escort Service Coimbatore
 
(👑VVIP ISHAAN ) Russian Call Girls Service Navi Mumbai🖕9920874524🖕Independent...
(👑VVIP ISHAAN ) Russian Call Girls Service Navi Mumbai🖕9920874524🖕Independent...(👑VVIP ISHAAN ) Russian Call Girls Service Navi Mumbai🖕9920874524🖕Independent...
(👑VVIP ISHAAN ) Russian Call Girls Service Navi Mumbai🖕9920874524🖕Independent...
 
Russian Escorts Girls Nehru Place ZINATHI 🔝9711199012 ☪ 24/7 Call Girls Delhi
Russian Escorts Girls  Nehru Place ZINATHI 🔝9711199012 ☪ 24/7 Call Girls DelhiRussian Escorts Girls  Nehru Place ZINATHI 🔝9711199012 ☪ 24/7 Call Girls Delhi
Russian Escorts Girls Nehru Place ZINATHI 🔝9711199012 ☪ 24/7 Call Girls Delhi
 

Arterial_thrombosis_and_embolism-1.pdf

  • 1. 1 Ministry of health of Ukraine Vinnytsya national Pirogov memorial medical university APPROVED on the methodical conference of department of endoscopic and cardiovascular surgery of medical faculty №1 Head of Department Professor ___________ V.V.Petrushenko. «______» ___________________20___ METHODICAL RECOMMENDATIONS for students Educational discipline Surgery Module № 2 Informative module 3 Theme of classes Arterial thrombosis and embolism. Classification of acute limb ischemia. Clinical stages of motion. Diagnostic. Differential diagnosis Methods of surgical treatment. Course V Department Medical №1 Vinnytsya 2016
  • 2. 2 Actuality Vascular diseases of extremities are widespread enough pathology with that in the first turn, surgeons, domestic and district doctors, and also the doctors of urgent first-aid have business mostly. Arterial thrombosis and embolisms of main vessels are a division of urgent surgery of vessels, which until now is to perfection an unsolved problem. It and large death rate (from 20 to 35%), large frequency of amputations of extremities on soil of gangrene, loss of capacity, a considerable exit on disability of patient, that worsens quality of life. The timely diagnostician of sharp arterial ischemia, reduction of time for diagnostics and grant of skilled medical care is stipulated by the economies of extremities and considerably improve the social problems of patient. Educational objectives: - to teach students to recognize the symptoms of acute arterial ischemia; - to able to differentiate embolism and thrombosis of blood vessels; - to be able to determine the location of vascular lesions; - to teach students to choose their own method of operative surgery; - to know the methods of surgical treatment of thrombosis, embolism; - to conduct necessary conservative therapy (thrombolytics, anticoagulants, antiplatelet agents, antispasmodics, etc.). The student should to know: - arterial and venous circulatory system; - to determination of acute arterial ischemia; - causes thrombosis; - morphological tissue changes in acute arterial obstruction; - reasons for the development of acute arterial ischemia; - to definition and causes of arterial embolism; - classification of acute arterial ischemia (V.S.Savelyev, 1978); - symptoms and clinical course of acute limb ischemia; - variants of clinical course and complications; - diagnostic algorithm examination of the patient; - differential diagnosis of thrombosis, embolism, spasm of the arteries of the extremities; - tactics and choice of treatment. The student should to be able to: - to identify pulsation in the extremities; - to identify violations of blood circulation;
  • 3. 3 - to diagnose the causes of artisanal embolism and thrombosis; - to identify the clinical signs of acute arterial ischemia; - to interpretation pursue complementary examination methods (coagulation, vascular ultrasound, angiography of blood vessels); - to able to determine the location of thrombus and embolism; - to be able to formulate a diagnosis violation artisanal circulation; - to conduct a differential diagnosis of thrombosis, embolism, spasm of the arteries of the extremities; - to be able to work out the tactics of choice of surgical treatment; - to hold anticoagulation or thrombolysis. Table of contents of theme Sharp arterial impassability Sharp arterial impassability believe the sudden cessation of blood circulation in the main artery embolism as a result of thrombosis or against other diseases. Etiology and pathogenesis In 80-93% of cases the cause arterial embolism have heart disease, including atherosclerotic infarction, acute myocardial infarction, rheumatic mitral defects like. When arterial embolism of the extremities due to lack of time the possibility of development of collateral circulation is always limited. Acute thrombosis prerequisites for enhancing collateral circulation in most cases formed during the progression of the underlying disease (atherosclerosis, arteritis, etc.). It also helps to increase the flow of arterial occlusion proximal level. Causes intravascular thrombus R. Virhov (1856), which united them-known triad: damage to the vascular wall, changes in the blood and circulatory disorders. In 90% of cases the cause of arterial thrombosis is "arteriosclerosis". Pathomorphism In patients with acute arterial obstruction in the first minute of watching histochemical and ultrastructural changes in tissue that indicate their ischemia: no cytochrome, succinate dehydrogenase, glycogen and degradation of existing mitochondria. In the future, there are degenerative, necrobiotic, necrotic processes, and infarcts, hemorrhage and gangrene. Classification of acute ischemic limb The classification is based by V.S.Savelyevym (1973) laid the clinical course of the disease (table 1).
  • 4. 4 Table 1 The degree of limb ischemia The main feature of IS (ischemic stress) No symptoms of ischemia at rest (their appearance only at load) I A Numbness, cold, paresthesia I B Pain II A Paresis II B Plehiya III A Subfastsialnyy swelling muscles III B Partial contracture III C Total contraction Symptomatology and clinical motion Subjective symptoms of acute ischemic limbs. The pain in the affected limb in most cases is not the first sign of acute arterial obstruction. Especially when he is pronounced embolism. The beginning of the disease in these cases is always sudden, although the power of pain can be different. However, acute thrombosis of major arteries syndrome pain in most cases develops gradually. In the initial stages of its intensity expressed slightly, then it increases, and thereafter it may no different from similar embolism. The nature of this syndrome is primarily caused by low arterial spasm and depends on the extent of ischemia tissue. Feeling of numbness, drop in temperature, paresthesia appear for patients both at embolism and at a sharp thrombosis. They usually grumble about feeling of "crawl of ants", passing of electric current" in extremity, "pricking needles". The degree of expression of the indicated complaints often depends on the psychological equilibrium of patients. Evidence of acute ischemic limbs. Change the color of the skin. At the onset of the disease observed moderate pale skin. Then it joins cyanosis, which is at a much severe ischemia causes the "marbled pattern" skin. Reducing the skin temperature celebrate almost 100% of cases. Its fluctuations acquire a wide range, so always have to measure the temperature in the symmetric parts of sick and healthy limb. Sensitivity disorders also constantly accompany acute ischemia of the limbs. In the initial stages of arterial occlusion reduced the surface, and then, with an increase ischemia, and deep sensitivity. During prolonged ischemia and significant comes complete anesthesia. Violations of active movements in joints limbs are characteristic of ischemia. They can occur either as constraints (paresis), or absence (plehiya) active movements or first in the distal and later in the proximal joints. Tenderness ischemic muscle indicates unfavorable course of acute ischemic
  • 5. 5 limbs and usually occurs in severe forms. Subfastsialnyy swelling of the muscles occurs when long-standing, long, very severe ischemia. It mainly "tight" and does not extend above the knee joint and the feet above the elbow to the hand. Coronary heart muscle contracture usually indicates the beginning necrobiotic phenomena in limbs. There are partial (partial) individual contracture of joints in which passive movements are not possible, and try to active movements painful, and total (complete) contracture, in which the movement is not possible in all joints. Clinical manifestations of ischemia are most pronounced in the distal limb. Their proximal limits are always below the occlusion, but define it in many cases impossible. Variants of clinical motion and complication Acute occlusion of the aortic bifurcation is characterized by clinical signs of ischemia both extremities, anterior abdominal wall, pelvic and distal spinal cord. Acute occlusion of the femoral artery, usually develops in the origin of the deep artery of the thigh. The upper limit is poor circulation between the upper and middle thirds of the thigh. In all cases, time has not treated the disease movements in the ankle joints of fingers and feet are missing, and only 28-30% of patients with this symptom can be observed in the knee joint. The sharp occlusion of popliteal artery mostly arises up at the level of trifurcation, that is on a 6-7 cm below poples. The top limit of violation of sensitiveness is no communicative between overhead and lower one third of shin. The changes of agile functions, as a rule, will keep indoors the joint of tibial foot for an area. For the occlusion of arteries of overhead extremities characteristic is that a sharp ischemia arises up in young age (35-40 years) and in default of any diseases of heart. In anamnesis on occasion it is possible to reduce the phenomenon of plexites or neck-pectoral radiculitis, quite often at the inspection of supraclavicular area find out smoothing or bulgingof supraclavicular fossula at palpation of that determine dense education with a pulsation. Classification of variants of motion of sharp ischemia of extremity Table 2 A sharp increasing ischemia of extremity A sharp ischemia of extremity is with the moderate stabilizing A sharp ischemia of extremity is with progress Ischemia of tension Ischemia of calmness With slow With rapid growth Quick as lightning With slow progress With rapid progress
  • 6. 6 growth Gangrene Chronic arterial insufficiency Semiotics angiographic embolism, breakage of contrast in the area of bifurcation of the arteries, "poor" net of collateral (Pict. 1). Angiographic semiotics of acute thrombosis, placed crooked line breakage contrast, uneven, jagged contours on fonf artery atherosclerotic vascular lesions and well-developed net of collateral (Pict. 2). Pict. 1. Embolism of femoral Pict. 2. Thrombosis of right artery. Arteriogram. A pointer general iliac artery. Arteriogram. is indicate localization of A pointer is indicate embolus. localization of blood clot.
  • 7. 7 Diagnostic program 1. Complaints,anamnesis. 2. Review of extremity, estimation of viability of ischemic tissues. 3. Palpation, auscultation of arteries. 4. Dopplerographic of inspection. 5. Aortoarteriohrafiya. 6. Coagulogram. The gangrene of extremity, that is necessary to be considered the most dangerous complication with a poor prognosis, develops in case of the unliquidated sharp arterial impassability. Differential diagnosis Table 3 Diagnostic sign Embolism Sharp thrombosis Spasm Basic disease Rheumatic defeats of heart, blinking arrhythmia. Atherosclerosis, trauma, malignant tumors. Heart disease, trauma, acute thrombophlebitis. Beginning Acute Subacute Acute Pain Very intense Mild Intense Ischemia Sharply expressed Moderate Moderate Skin sensitivity Anesthesia Reduced Reduced Motor function Ischemic paralysis Reduced muscle strength Reduced muscle strength Skin temperature Sharply reduced Moderately reduced Moderately reduced The color of the skin Pale, almost white or marble Pale Pale purple tint Swelling of limbs Sometimes, after first aid and only shin Do not have There is early in acute venous thrombosis
  • 8. 8 Angiography A clear contrast level cliff The contours of the vessel unequal, line break Vessels with smooth disappearance of contrast Collaterals Missing A large number of Collaterals of narrowed diameter Tactics and choice of treatment In acute arterial obstruction indications for conservative treatment there can only be a contraindication to surgery. Absolute contraindications are: agonal condition of the patient, total limb ischemic contracture (acute ischemia degree IPR), a very serious condition patient with mild ischemia (acute ischemia IA - IB degrees). By relative contraindications should include acute myocardial miokardachy stroke, tumors that are not subject to the operation, and easy ischemia without signs of progression. In all other cases of acute arterial obstruction appropriate emergency surgery. The optimal time for surgery for embolism is the first 6 hours of onset and acute thrombosis - 12-24 hours. The essence of surgery (Pict. 3) embolism is emboli generally remove the catheter, vascular rings, vacuum suction, by retrograde flushing the arteries Pict. 4), or spurly balloon probe. The most famous among them, recently received a special method embolectomy balloon catheter by Fogarty. Its use has made low- impact operation, simple and much more effective. Surgeries acute thrombosis fundamentally different from dealing with embolism. Features of surgical intervention in patients with acute thrombosis is that while thrombectomy should also eliminate the cause of the disease, that performs particular reconstruction arteries. In uncomplicated postoperative patients can prescribe outpatient treatment for 12-15 days. In the future, they should be at the dispensary under the control of the surgeon, and if necessary - and cardiologist and rheumatologist. After undergoing reconstructive surgery on main arteries they must also conduct periodic state determination coagulation of the blood system. Such patients should receive aspirin regularly and trental. If necessary, we must purposefully use indirect anticoagulants with obligatory once in six months, a full examination of the patient in the clinic or hospital conditions.
  • 9. 9 A B C D Pict. 3. Removal of emboli (scheme): A - stream of blood; B - tweezers; C - Fogarty catheter; D - vacuum suction. Pict.4. Scheme of operation of the retrograde washing of artery force-feeds. Tests and tasks are added.
  • 10. 10 Theme: The arterial thrombosis and embolisms Tests (Step-2) Variant 1 1. The patient, 35 years old, from rheumatism in the active phase has mitral valve stenosis fourth stage, atrial fibrillation. Six hours ago the expressed pains appeared at a calmness in a right lower limb, at an objective review the arterial pulsation of femoral artery is absent on the right, the temperature of skin of foot is mionectic, bluish color, motion limits in a foot, a sensitiveness is stored. Define illness. A. Sharp violation of cerebral circulation of blood B. Sharp right femoral artery thromboembolism C. Sharp thrombophlebitis of deep veins of right shin D. Erysipelas right tibia E. Lymphangoitis acute right lower limb 2. A patient with the sharp ischemia of both lower limbs, that arose up simultaneously, absent pulsation on right and left femoral, popliteal arteries. Great pain appeared in a stomach, vomit, collapse, liquid chairs, positive symptoms of irritation of peritoneum. What is the condition of the patient? A. Occlusion of right iliac artery B. Thrombosis and embolism aortic bifurcation inferior mesenteric artery C. Thrombosis of both femoral arteries D. Layering aortic aneurysm E. Total thrombosis of bifurcation of aorta 3. The man complained of pain in the left arm, pain at bending in elbow and radial-wrist joints. Pains appeared 12 hours ago after heavy physical activity. Objectively: the edema of the left brush, forearm, shoulder is expressed. Skin on the left limb cyanotic, reduced pain and tactile sensitivity. Pulse on the hand, forearm and shoulder is not defined. Formulate a preliminary diagnosis: A. Sharp thrombosis of the subclavian vein B. Sharp thrombosis of humeral artery C. Syndrome by Reyno D. Illness by Reyno E. Break of armpit artery
  • 11. 11 4. The patient, 56 years old, complaining of intense pain in the left leg, there were three hours ago suddenly, numbness and cold extremities. During the year, noted the pain in the leg while walking, increased sensitivity to cooling. The left foot and lower leg with marble pattern, marked "a symptom of the groove." At the touch of foot cold reduced active movement of the foot, toes saved. Pulse detectable only in the femoral artery. Above her systolic murmur is heard. Formulate a preliminary diagnosis: A. Endarteritis obliterans B. Syndrome by Reyno C. Sharp thrombophlebitis D. Atherosclerosis of vessels of lower limbs, sharp thrombosis E. Illness by Buerger 5. The patient, 62 years old, complaining of intense pain in the left leg, there were three hours ago suddenly, numbness and cold extremities. During the year, noted the pain in the leg while walking, increased sensitivity to cooling. The left foot and lower leg with marble pattern, saphenas slept. At the touch of foot cold reduced active movement of the foot, toes saved. Pulse detectable only in the femoral artery. Above it rough systolic murmur is heard. Formulate a preliminary diagnosis: A. Endarteritis obliterans B. Stenosis of the left popliteal artery C. Sharp occlusion of the left femoral artery D. Sharp thrombophlebitis E. Sharp ileofemoralnyy arterial thrombosis
  • 12. 12 Theme: The arterial thrombosis and embolisms Tests (Step-2) Variant 2 1. In patients with acute ischemia of the extremities of both encountered while, but has more significant manifestations of matter, there is no ripple in the right femoral, popliteal arteries and the arteries of the foot. Determined relaxation ripple left thigh arteries and other arteries of the left lower limb. Determine the level of occlusion: A. Occlusion of the right iliac artery B. Tromb - "rider" aortic bifurcation C. Occlusion of both femoral arteries D. Forensic thrombosis aortic bifurcation E. Occlusion of the right external iliac and femoral arteries of the left internal 2. The patient, 45 years old, is on treatment for rheumatism, active phase, combined mitral valve defect. During the morning the toilet suddenly felt pain in his left hand, followed her numbness. Pain and numbness are grew. Objective: pale skin of his left hand, relatively cold. Pulsation of the arteries throughout the hand is missing. What is the most appropriate therapeutic approach? A. Appointment fibrinolysis, anticoagulants B. Purpose of antibiotics and anti-inflammatory drugs C. Emergency embolectomy D. Probing heart E. Emergency trombintymektomiya 3. In the district hospital which is located 30 km from the central district hospital and 180 km from the regional hospital, hospitalized patient is 62 years of intramural myocardial infarction. After 25 days of onset of night he suddenly appeared sharp pain in the left lower limb. Ending immediately became pale. Gone active movement of toes and ankle-stage joint, decreased skin sensitivity to palpation of the calf muscle moderately painful. Ripple in the hip during inguinal artery connection, left - enhanced, compared with femoral artery pulsation right. What is the most likely diagnosis?
  • 13. 13 A. Thrombosis of the femoral artery B. Atherosclerosis of the arteries of the lower extremities C. Embolism femoral artery D. Illness by Reyno E. Endarteritis obliterans of the lower extremities 4. The patient, 60 years old, there was a sharp pain in the left lower limb, which appeared within hours and growing. Vascular Angiography revealed that the contours of the popliteal artery uneven, irregular line break, plenty of collaterals. What is the status of the patient? A. Popliteal artery embolism B. Trumbo popliteal artery C. Spasm of the popliteal artery D. Illness by Reyno E. Tthrombophlebitis 5. The patient came to the clinic 3 hours ago with the onset of pain in his right leg. Sick 10 years, suffering from atherosclerosis. The end of the cold to the touch, pale, bluish tint of the foot and lower leg. Ripple on the right femoral artery is absent. What is the preliminary diagnosis? A. Sharp thromboangiitis right foot B. Deep vein thrombosis sharp right foot C. Sharp right femoral artery spasm D. Sharp thrombosis of the right femoral artery E. Sharp lymphocytosis, lymphedema
  • 14. 14 Theme: The arterial thrombosis and embolisms Tests (Step-2) Variant 3 1. The patient, 76 years old, ambulance transported in serious condition with complaints of sharp pain in the left lower limb, which appeared within hours and growing. When walking feels the lack of solid support, cold feet. He suffers from coronary artery disease, diabetes mellitus. On examination, the left lower limb pale knee, shin and foot cold under the skin traced small vessels. Pain sensitivity broken leg and foot on the left. Ripple on foot and popliteal fossa missing left, weakened femoral arteries. A. Popliteal artery thrombosis B. Diabetic gangrene of the left lower limb C. Atherosclerosis D. Illness by Reyno E. Thrombophlebitis 2. In the department of vascular surgery delivered from the central district hospital patient, 66 years old, with complaints of pain, loss of sensitivity and movement in his right foot. These symptoms arose three days ago, was treated conservatively. Objective: body temperature - 38.2 degrees, skin pale, blood pressure - 110/70 mm of mercury column, pulse - 110 min., Locally - foot and leg cold, active movement in the knee and foot are absent, swelling and soreness of muscles legs, heart rate is determined only on the femoral artery. What should to be the surgeon? A. Conduct of the popliteal artery thrombectomy B. Make fastsiotomiya on the shin C. Hold thrombolytic therapy with streptokinase D. Do amputation at the hip E. Do amputation at the tibia 3. The man complained of pain in the left arm, pain in bending the elbow and Acute radiation-carpal bones joints. Pain appeared 12 hours ago after heavy exercise. Objective: edema of the left hand, forearm, shoulder. Skin on the left limb cyanotic, reduced pain and tactile sensitivity. Pulse on the hand, forearm and shoulder is not defined. Formulate a preliminary diagnosis:
  • 15. 15 A. Sharp thrombosis of the subclavian vein B. Sharp brachial artery thrombosis C. Syndrome by Reyno D. Illness by Reyno E. The gap groin artery 4. The patient was hospitalized in the department of vascular surgery on the third day of onset with severe muscular contracture of the right leg. Leg limited in all joints mobile, bluish skin color, reaction to pain stimulation there. What is type of surgery need to do? A. Embolectomy B. Tromboektomiya C. Amputation D. Shunting E. Prosthesis 5. In the district hospital which is located 30 km from the central district hospital and 180 km from the regional hospital, hospitalized patient is 62 years of intramural myocardial infarction. After 25 days of onset of night he suddenly appeared sharp pain in the left lower limb. Ending immediately became pale. Gone active movement of toes and ankle-stage joint, decreased skin sensitivity to palpation of the calf muscle moderately painful. Ripple in the hip during inguinal artery connection, left - enhanced, compared with femoral artery pulsation right. What is the most likely diagnosis? A. Thrombosis of the femoral artery B. Atherosclerosis of the arteries of the lower extremities C. Embolism femoral artery D. Illness by Reyno E. Endarteritis obliterans of the lower extremities
  • 16. 16 Theme: The arterial thrombosis and embolisms Tests (Step-2) Variant 4 1. The patient, 52 years old, complained of sharp pain in his left leg and foot, which came 2 hours ago when you change position in bed? Sick mitral heart defect and atrial fibrillation. Locally: left foot and lower leg pale, cold to the touch, active movement in the foot limited tactile sensitivity of the limbs is reduced. Ripple in the popliteal artery and below the missing on both femoral and popliteal right - preserved. What is causes this pathology clinical picture? A. Sharp thrombosis of the left femoral artery B. Spasm of artery left lower limb C. Atherosclerosis left lower limb D. Left popliteal artery embolism E. Lymphocytosis left lower limb 2. In patients suffering from heart defect and atrial fibrillation, sudden sharp pain in the left lower leg and foot. Foot and lower third of the tibia pale, cold to the touch. Palpation shin painful tactile sensitivity is reduced, limited movement, pulsation of the arteries of the foot is not defined. What is the most likely diagnosis? A. Sharp thrombosis B. Sharp phlebitis C. Erysipelas D. Thrombosis of the femoral artery E. Gangrene 3. The patient is taken to hospital after 1 hour after the onset of severe pain in his left leg. In history - heart defect. The left foot is cold to the touch, pale thigh sharply, palpation patient complains of sharp pain. Ripple femoral artery during under Pouparts ligament expressive; in other arteries of the legs - no pulsation. Formulate a preliminary diagnosis: A. Sharp thrombosis left leg B. Deep vein thrombosis, sharp left leg C. Sharp thrombosis of the left femoral artery D. Sharp lymphocytosis, lymphedema left
  • 17. 17 E. Sharp left shin phlebothrombosis 4. The patient, 42 years old, who suffers from rheumatism and combined mitral heart defect suddenly appeared sharp pain in the right upper extremity for 2 hours. In vascular angiographic examination revealed a clear contrast level of breakage of the right brachial artery collaterals available. What is the status of the patient? A. Brachial artery thrombosis B. Spasm of the brachial artery C. Brachial artery embolism D. Illness by Reyno 5. The patient, 35 years old, from rheumatism in the active phase has mitral valve stenosis fourth stage, atrial fibrillation. Six days ago there were severe pain at rest in the right lower limb, with no objective review of arterial pulsation of the femoral artery right, foot skin temperature is lowered, bluish color, foot movement is limited, sensitivity is preserved. Identify disease. A. Sharp cerebrovascular accident B. Sharp right femoral artery thromboembolism C. Sharp deep vein thrombosis right leg D. Erysipelas right tibia E. Lymphangoitis sharp right lower limb
  • 18. 18 Theme: The arterial thrombosis and embolisms Tests (Step-2) Variant 5 1. The patient, 32 years old, who suffers congenital heart defect and atrial fibrillation, sudden sharp pain in the left lower leg and foot. The patient was examined in an hour at home. She moans in pain, foot and lower third of the tibia pale, cold. Palpation sharply painful shin mobility in the ankle joint is absent, superficial and deep sensitivity on the feet are missing. Pulse on femoral arteries clear, the popliteal artery and arteries of the foot - no. What did disease appeared in the patient? A. Left femoral artery thrombosis B. Sharp left popliteal artery embolism. Ischemia II B degrees. Sharp deep vein thrombosis of the left lower limb C. Aftertromboflebitychnyy left lower limb syndrome D. Atherosclerosis of the left lower limb. Occlusion of the femoral- segment hamstring 2. The patient was hospitalized in the department of vascular surgery three days from the onset of severe muscular contracture of the right leg. Leg limited in all joints mobile, bluish skin color, reaction to pain stimulation there. What is a preliminary diagnosis? A. Sharp arterial thrombosis B. Sharp phlebothrombosis C. Contracture by Dupuytren D. Sharp thrombosis E. Position syndrome 3. The patient, 35 years old, from rheumatism in the active phase has mitral valve stenosis fourth stage, atrial fibrillation. Six hours ago the expressed pains appeared at a calmness in a right lower limb, at an objective review the arterial pulsation of femoral artery is absent on the right, the temperature of skin of foot is mionectic, bluish color, motion limits in a foot, a sensitiveness is stored.
  • 19. 19 A. ECG B. Ultrasound of the heart C. Extremity vascular ultrasound D. Monitoring by Holter E. Phlebography 4. In patients with acute ischemia of the extremities of both encountered while, but has more significant manifestations of matter, there is no ripple in the right femoral, popliteal arteries and the arteries of the foot. Determined relaxation ripple left thigh arteries and other arteries of the left lower limb. Determine the level of occlusion: F. Occlusion of the right iliac artery G. Tromb - "rider" aortic bifurcation H. Occlusion of both femoral arteries I. Forensic thrombosis aortic bifurcation J. Occlusion of the right external iliac and femoral arteries of the left internal 5. The boy, 10 years old, incised wound of the anterior forearm. With wounds stems thin trickle of dark blood. What method should temporarily stop bleeding apply? A. Finger pressing brachial artery B. Apply braid above the wound C. Apply bandages shaking D. Apply braid wound below E. Use the method to overbend limbs
  • 20. 20 Theme: The arterial thrombosis and embolisms Tests (Step-2) Variant 6 1. The patient, 62 years old, complaining of intense pain in the left leg, there were three hours ago suddenly, numbness and cold extremities. During the year, noted the pain in the leg while walking, increased sensitivity to cooling. The left foot and lower leg with marble pattern, saphenas slept. At the touch of foot cold reduced active movement of the foot, toes saved. Pulse detectable only in the femoral artery. Above it rough systolic murmur is heard. Formulate a preliminary diagnosis: A. Endarteritis obliterans B. Stenosis of the left popliteal artery C. Sharp occlusion of the left femoral artery D. Sharp thrombophlebitis E. Sharp ileofemoralnyy arterial thrombosis 2. A patient suffering from combined mitral defect for 15 years. 6 hours ago there were severe pain in the left leg. On examination, the left foot is cold to the touch, pale from the foot to the thigh, painful on palpation, pulsation of the femoral artery is determined only by Pouparts connection, other arteries no pulse. What is a preliminary diagnosis? A. Sharp thrombosis of the left femoral artery B. Sharp trombovlebit superficial veins of the left thigh C. Deep vein thrombosis, sharp left hip D. Sharp lymph venous stasis left foot E. Aneurysm of the left femoral artery 3. Patient 5 hours ago was in a car accident. He complains of abdominal pain, headache. Blood pressure was 180/100 mmHg. In Plain radiography free abdominal organs no fluid in the abdominal flank were found. Severe oliguria, urine output of 10 mL / hour. In the abdomen on the white line, largely left palpable formation 10h8 cm., tautoelastic consistency over which auscultation systolic murmur is heard. Over the last 2:00 noticeable trend towards larger entities. Symptoms of peritoneal irritation there. Put the preliminary diagnosis:
  • 21. 21 A. Traumatic aneurysm of the abdominal aorta B. Two-step subscapular spleen rupture C. Retroperitoneal hematoma D. Rupture of liver subscapular hematoma E. Post-traumatic hematoma ripples small intestine 4. In patients with acute ischemia of the extremities of both encountered while, but has more significant manifestations of matter, there is no ripple in the right femoral, popliteal arteries and the arteries of the foot. Determined relaxation ripple left thigh arteries and other arteries of the left lower limb. What is a complementary method to assign? A. Vascular ultrasound B. Angiography of aorta C. Phlebography D. Ultrasound of the heart 5. For a patient with the sharp ischemia of both lower limbs, that arose up simultaneously, absent pulsation on right and left femoral, popliteal arteries. Great pain appeared in a stomach, vomit, collapse, liquid chairs, positive symptoms of irritation of peritoneum. What is the condition of the patient? A. Occlusion of right iliac artery B. Thrombosis and embolism aortic bifurcation inferior mesenteric artery C. Thrombosis of both femoral arteries D. Layering aortic aneurysm E. Total thrombosis of bifurcation of aorta
  • 22. 22 Theme: The arterial thrombosis and embolisms Tests (Step-2) Variant 7 1. The patient came to the clinic 3 hours ago with the onset of pain in his right leg. Sick 10 years, suffering from atherosclerosis. The end of the cold to the touch, pale, bluish tint of the foot and lower leg. Ripple on the right femoral artery is absent. What is the preliminary diagnosis? A. Sharp thromboangiitis right foot B. Deep vein thrombosis sharp right foot C. Sharp right femoral artery spasm D. Sharp thrombosis of the right femoral artery E. Sharp lymphocytosis, lymphedema 2. The patient, 45 years old, is on treatment for rheumatism, active phase, combined mitral valve defect. During the morning the toilet suddenly felt pain in his left hand, followed her numbness. Pain and numbness are grew. Objective: pale skin of his left hand, relatively cold. Pulsation of the arteries throughout the hand is missing. What is the most appropriate therapeutic approach? A. Appointment fibrinolysis, anticoagulants B. Purpose of antibiotics and anti-inflammatory drugs C. Emergency embolectomy D. Probing heart E. Emergency trombintymektomiya 3. The boy, 10 years old, incised wound of the anterior forearm. With wounds stems thin trickle of dark blood. What method should temporarily stop bleeding apply? A. Finger pressing brachial artery B. Apply braid above the wound C. Apply bandages shaking D. Apply braid wound below E. Use the method to overbend limbs
  • 23. 23 4. The patient, 32 years old, who suffers congenital heart defect and atrial fibrillation, sudden sharp pain in the left lower leg and foot. The patient was examined in an hour at home. She moans in pain, foot and lower third of the tibia pale, cold. Palpation sharply painful shin mobility in the ankle joint is absent, superficial and deep sensitivity on the feet are missing. Pulse on femoral arteries clear, the popliteal artery and arteries of the foot - no. What did disease appeared in the patient? A. Left femoral artery thrombosis B. Sharp left popliteal artery embolism. Ischemia II B degrees. Sharp deep vein thrombosis of the left lower limb C. Aftertromboflebitychnyy left lower limb syndrome D. Atherosclerosis of the left lower limb. Occlusion of the femoral- segment hamstring. 5. In the department of vascular surgery delivered from the central district hospital patient, 66 years old, with complaints of pain, loss of sensitivity and movement in his right foot. These symptoms arose three days ago, was treated conservatively. Objective: body temperature - 38.2 degrees, skin pale, blood pressure - 110/70 mm of mercury column, pulse - 110 min., Locally - foot and leg cold, active movement in the knee and foot are absent, swelling and soreness of muscles legs, heart rate is determined only on the femoral artery. What should to be the surgeon? A. Conduct of the popliteal artery thrombectomy B. Make fastsiotomiya on the shin C. Hold thrombolytic therapy with streptokinase D. Do amputation at the hip E. Do amputation at the tibia
  • 24. 24 Theme: The arterial thrombosis and embolisms Tests (Step-2) Variant 8 1. A patient with the sharp ischemia of both lower limbs, that arose up simultaneously, absent pulsation on right and left femoral, popliteal arteries. Great pain appeared in a stomach, vomit, collapse, liquid chairs, positive symptoms of irritation of peritoneum. What is the condition of the patient? A. Occlusion of right iliac artery B. Thrombosis and embolism aortic bifurcation inferior mesenteric artery C. Thrombosis of both femoral arteries D. Layering aortic aneurysm E. Total thrombosis of bifurcation of aorta 2. The man complained of pain in the left arm, pain in bending the elbow and Acute radiation-carpal bones joints. Pain appeared 12 hours ago after heavy exercise. Objective: edema of the left hand, forearm, shoulder. Skin on the left limb cyanotic, reduced pain and tactile sensitivity. Pulse on the hand, forearm and shoulder is not defined. Formulate a preliminary diagnosis: A. Sharp thrombosis of the subclavian vein B. Sharp brachial artery thrombosis C. Syndrome by Reyno D. Illness by Reyno E. The gap groin artery 3. In the district hospital which is located 30 km from the central district hospital and 180 km from the regional hospital, hospitalized patient is 62 years of intramural myocardial infarction. After 25 days of onset of night he suddenly appeared sharp pain in the left lower limb. Ending immediately became pale. Gone active movement of toes and ankle-stage joint, decreased skin sensitivity to palpation of the calf muscle moderately painful. Ripple in the hip during inguinal artery connection, left - enhanced, compared with femoral artery pulsation right. What is the most likely diagnosis? A. Thrombosis of the femoral artery B. Atherosclerosis of the arteries of the lower extremities C. Embolism femoral artery D. Illness by Reyno
  • 25. 25 E. Endarteritis obliterans of the lower extremities 4. The patient, 62 years old, complaining of intense pain in the left leg, there were three hours ago suddenly, numbness and cold extremities. During the year, noted the pain in the leg while walking, increased sensitivity to cooling. The left foot and lower leg with marble pattern, saphenas slept. At the touch of foot cold reduced active movement of the foot, toes saved. Pulse detectable only in the femoral artery. Above it rough systolic murmur is heard. Formulate a preliminary diagnosis: A. Endarteritis obliterans B. Stenosis of the left popliteal artery C. Sharp occlusion of the left femoral artery D. Sharp thrombophlebitis E. Sharp ileofemoralnyy 5. The patient, 76 years old, ambulance transported in serious condition with complaints of sharp pain in the left lower limb, which appeared within hours and growing. When walking feels the lack of solid support, cold feet. He suffers from coronary artery disease, diabetes mellitus. On examination, the left lower limb pale knee, shin and foot cold under the skin traced small vessels. Pain sensitivity broken leg and foot on the left. Ripple on foot and popliteal fossa missing left, weakened femoral arteries. A. Popliteal artery thrombosis B. Diabetic gangrene of the left lower limb C. Atherosclerosis D. Illness by Reyno E. Thrombophlebitis
  • 26. 26 Theme: The arterial thrombosis and embolisms Tests (Step-2) Variant 9 1. The boy, 10 years old, incised wound of the anterior forearm. With wounds stems thin trickle of dark blood. What method should temporarily stop bleeding apply? A. Finger pressing brachial artery B. Apply braid above the wound C. Apply bandages shaking D. Apply braid wound below E. Use the method to overbend limbs 2. The patient was hospitalized in the department of vascular surgery on the third day of onset with severe muscular contracture of the right leg. Leg limited in all joints mobile, bluish skin color, reaction to pain stimulation there. What is type of surgery need to do? A. Embolectomy B. Tromboektomiya C. Amputation D. Shunting E. Prosthesis 3. In patients suffering from heart defect and atrial fibrillation, sudden sharp pain in the left lower leg and foot. Foot and lower third of the tibia pale, cold to the touch. Palpation shin painful tactile sensitivity is reduced, limited movement, pulsation of the arteries of the foot is not defined. What is the most likely diagnosis? A. Sharp thrombosis B. Sharp phlebitis C. Erysipelas D. Thrombosis of the femoral artery E. Gangrene 4. The patient, 42 years old, who suffers from rheumatism and combined mitral heart defect suddenly appeared sharp pain in the right upper extremity for 2 hours. In vascular angiographic examination revealed a clear contrast level
  • 27. 27 of breakage of the right brachial artery collaterals available. What is the status of the patient? A. Brachial artery thrombosis B. Spasm of the brachial artery C. Brachial artery embolism D. Illness by Reyno 5. In the department of vascular surgery delivered from the central district hospital patient, 66 years old, with complaints of pain, loss of sensitivity and movement in his right foot. These symptoms arose three days ago, was treated conservatively. Objective: body temperature - 38.2 degrees, skin pale, blood pressure - 110/70 mm of mercury column, pulse - 110 min., Locally - foot and leg cold, active movement in the knee and foot are absent, swelling and soreness of muscles legs, heart rate is determined only on the femoral artery. What should to be the surgeon? A. Conduct of the popliteal artery thrombectomy B. Make fastsiotomiya on the shin C. Hold thrombolytic therapy with streptokinase D. Do amputation at the hip E. Do amputation at the tibia
  • 28. 28 Theme: The arterial thrombosis and embolisms Tests (Step-2) Variant 10 1. The patient, 60 years old, there was a sharp pain in the left lower limb, which appeared within hours and growing. Vascular Angiography revealed that the contours of the popliteal artery uneven, irregular line break, plenty of collaterals. What is the status of the patient? A. Popliteal artery embolism B. Trumbo popliteal artery C. Spasm of the popliteal artery D. Illness by Reyno E. Tthrombophlebitis 2. The patient, 62 years old, complaining of intense pain in the left leg, there were three hours ago suddenly, numbness and cold extremities. During the year, noted the pain in the leg while walking, increased sensitivity to cooling. The left foot and lower leg with marble pattern, saphenas slept. At the touch of foot cold reduced active movement of the foot, toes saved. Pulse detectable only in the femoral artery. Above it rough systolic murmur is heard. Formulate a preliminary diagnosis: A. Endarteritis obliterans B. Stenosis of the left popliteal artery C. Sharp occlusion of the left femoral artery D. Sharp thrombophlebitis E. Sharp ileofemoralnyy arterial thrombosis 3. The patient, 56 years old, complaining of intense pain in the left leg, there were three hours ago suddenly, numbness and cold extremities. During the year, noted the pain in the leg while walking, increased sensitivity to cooling. The left foot and lower leg with marble pattern, marked "a symptom of the groove." At the touch of foot cold reduced active movement of the foot, toes saved. Pulse detectable only in the femoral artery. Above her systolic murmur is heard. Formulate a preliminary diagnosis: A. Endarteritis obliterans B. Syndrome by Reyno C. Sharp thrombophlebitis
  • 29. 29 D. Atherosclerosis of vessels of lower limbs, sharp thrombosis E. Illness by Buerger 4. In patients with acute ischemia of the extremities of both encountered while, but has more significant manifestations of matter, there is no ripple in the right femoral, popliteal arteries and the arteries of the foot. Determined relaxation ripple left thigh arteries and other arteries of the left lower limb. What is a complementary method to assign? A. Vascular ultrasound B. Angiography of aorta C. Phlebography D. Ultrasound of the heart 5. The patient, 32 years old, who suffers congenital heart defect and atrial fibrillation, sudden sharp pain in the left lower leg and foot. The patient was examined in an hour at home. She moans in pain, foot and lower third of the tibia pale, cold. Palpation sharply painful shin mobility in the ankle joint is absent, superficial and deep sensitivity on the feet are missing. Pulse on femoral arteries clear, the popliteal artery and arteries of the foot - no. What did disease appeared in the patient? A. Left femoral artery thrombosis B. Sharp left popliteal artery embolism. Ischemia II B degrees. Sharp deep vein thrombosis of the left lower limb C. Aftertromboflebitychnyy left lower limb syndrome D. Atherosclerosis of the left lower limb. Occlusion of the femoral- segment hamstring
  • 30. 30 Theme: The arterial thrombosis and embolisms Tests (Step-2) Variant 11 1. The patient, 52 years old, complained of sharp pain in his left leg and foot, which came 2 hours ago when you change position in bed? Sick mitral heart defect and atrial fibrillation. Locally: left foot and lower leg pale, cold to the touch, active movement in the foot limited tactile sensitivity of the limbs is reduced. Ripple in the popliteal artery and below the missing on both femoral and popliteal right - preserved. What is causes this pathology clinical picture? A. Sharp thrombosis of the left femoral artery B. Spasm of artery left lower limb C. Atherosclerosis left lower limb D. Left popliteal artery embolism E. Lymphocytosis left lower limb 2. The patient is taken to hospital after 1 hour after the onset of severe pain in his left leg. In history - heart defect. The left foot is cold to the touch, pale thigh sharply, palpation patient complains of sharp pain. Ripple femoral artery during under Pouparts ligament expressive; in other arteries of the legs - no pulsation. Formulate a preliminary diagnosis: A. Sharp thrombosis left leg B. Deep vein thrombosis, sharp left leg C. Sharp thrombosis of the left femoral artery D. Sharp lymphocytosis, lymphedema left E. Sharp left shin phlebothrombosis 3. The patient, 76 years old, ambulance transported in serious condition with complaints of sharp pain in the left lower limb, which appeared within hours and growing. When walking feels the lack of solid support, cold feet. He suffers from coronary artery disease, diabetes mellitus. On examination, the left lower limb pale knee, shin and foot cold under the skin traced small vessels. Pain sensitivity broken leg and foot on the left. Ripple on foot and popliteal fossa missing left, weakened femoral arteries. A. Popliteal artery thrombosis B. Diabetic gangrene of the left lower limb
  • 31. 31 C. Atherosclerosis D. Illness by Reyno E. Thrombophlebitis 4. The patient was hospitalized in the department of vascular surgery three days from the onset of severe muscular contracture of the right leg. Leg limited in all joints mobile, bluish skin color, reaction to pain stimulation there. What is a preliminary diagnosis? A. Sharp arterial thrombosis B. Sharp phlebothrombosis C. Contracture by Dupuytren D. Sharp thrombosis E. Position syndrome 5. The patient, 35 years old, from rheumatism in the active phase has mitral valve stenosis fourth stage, atrial fibrillation. Six days ago there were severe pain at rest in the right lower limb, with no objective review of arterial pulsation of the femoral artery right, foot skin temperature is lowered, bluish color, foot movement is limited, sensitivity is preserved. Identify disease. A. Sharp cerebrovascular accident B. Sharp right femoral artery thromboembolism C. Sharp deep vein thrombosis right leg D. Erysipelas right tibia E. Lymphangoitis sharp right lower limb